You are on page 1of 44

What may be insured against, contingencies – 3, 89, 182-183, Sec.

3, Rule 131 defendant, was undoubtedly an accident insurance, as distinguished from a life
Rules of Court insurance. As conceded by the facts appearing in the pleadings, the personal
EN BANC accident policy, part of the proceeds of which is under garnishment, was for
G.R. No. L-12189 April 29, 1960 P50,000.00 and yet the annual premium was for P15.00. If it were an ordinary life
FRANCISCA GALLARDO, plaintiff-appellee, insurance policy, taking into account that the insured, Luis G. Morales, was 38 years
vs. of age and the amount of the policy was for P50,000.00 the annual premium would
HERMENEGILDA S. MORALES, defendant-appellant. have been around P1,206.00. Besides, the period for the policy was stipulated for
Cajulis and Dolorfino for appellee. one year, and considerations as to age, health, occupation and other personal
Filemon Cajator for appellant. circumstances were not taken into account in an accident insurance policy. Even the
CONCEPCION, J.: certification issued by the insurance commissioner on August 23, 1956, marked as
The issue before us is whether a personal accident insurance which "insures for Annex "1" of the opposition, shows that the Capital Insurance and Surety Company
injuries and/or death as a result of murder or assault or attempt thereat" is a life Inc. is a non-life insurance company and that the only authority granted to it to
insurance, within the purview of Rule 39, section 12, subdivision (k) of the Rules of transact business covers fire, marine, surety, fidelity, accident, motor car, and
Court, exempting from execution. miscellaneous insurance, except life insurance. From this circumstance alone, not to
All moneys, benefits, privileges, or annuities accruing or in any manner growing out mention many others, there are abundant indications that there exists a
of any life insurance, if the annual premiums paid do not exceed five hundred fundamental distinction between life insurance and accident insurance. As counsel
pesos, and if they exceed that sum a like exemption shall exist which shall bear the for oppositor has clearly pointed out, an accident policy merely insures the person
same proportion to the moneys, benefits, privileges, and annuities so accruing or from injury and or death resulting from murder, assault, or an attempt thereat,
growing out of such insurance that said five hundred pesos bears to the whole while in life insurance policy, what is insured is the life of the subject for a definite
annual premiums paid. number of years. From the authorities quoted by the oppositor, this Court is fully
In accordance with a compromise agreement between the parties in the above- convinced that an accident policy is fundamentally different from a life insurance
entitled case, a decision was rendered therein by the Court of First Instance of policy, especially if this Court takes into account that accident insurance is an
Manila, on February 3, 1956, sentencing defendant Hermenegilda S. Morales to pay indemnity or casualty contract, while life insurance is an investment contract.
to plaintiff Francisca Gallardo the sum of Seven Thousand Pesos (P7,000.00). In due It is not disputed that a life insurance is, generally speaking, distinct and different
course, the corresponding writ of execution was issued and delivered to the Sheriff from an accident insurance. However, when one of the risks insured in the latter is
of Manila, who, on August 8, 1956, garnished and levied execution on the sum of the death of the insured by accident, then there are authorities to the effect that
P7,000.00, out of the P30,000.00 a due from the Capital Insurance & Surety Co., such accident insurance may, also, be regarded as a life insurance.
Inc., to said defendant, as beneficiary under a personal accident policy issued by "Life insurance" is a contract whereby one party insures a person against loss by the
said company to defendant's husband, Luis Morales, who died, on August 26, 1950, death of another. Petition of Robbins, 140 A. 366, 367, 126 Me. 555.
by assassination. Invoking the above-quoted provision of the Rules of Court, An insurance on life is a contract by which the insurer, for a stipulated sum, engages
defendant asked the sheriff to quash and lift said garnishment or levy on execution. to pay a certain amount of money if another dies within the time limited by the
Upon denial of this request by the sheriff, defendant filed a motion praying that the policy. Cason vs. Owens, 26 S. E. 75, 76, 100 Ga. 142.
aforementioned sum of P7,000.00 be declared exempt from execution under said Life insurance includes in which the payment of the insurance money is contingent
provision of the Rules of Court, and that the Sheriff of Manila be ordered to quash upon the loss of life. Bowless vs. Mutual Ben. Health & Accident Ass'n, C.C.A. Va.
or lift said garnishment or levy on execution. This motion was denied by an order 99F. 2d 44. 48, 49.
dated October 18, 1956. Hence, the present appeal by the defendant, who A contract for life insurance is really a contract for insurance for one year in
maintains that the policy in question is a life insurance policy, within the purview of consideration of an advanced premium, with the right of assured to continue it
the aforementioned exemption, for it insured her husband ". . . for injuries and/or from year to year upon payment of a premium as stipulated. Mutual Life Ins. Co.
death as a result of murder or assault or attempt thereat." 100 Pa 172, 180.
In its order denying the claim for exemption set up by the defendant, the lower In its broader sense, "life insurance" includes accident insurance, since life is
court expressed itself as follows: insured under either contract. American Trust & Banking Co. vs. Lessly, 106 S.W. 2d.
Upon a perusal of the authorities cited by the parties, this Court is fully convinced 551, 552, 171 Tenn. 561, 111 A.L.R. 59.
that there is a fundamental distinction between life insurance, and accident Under statute providing that 'any life insurance' on life of husband shall insure to
insurance, and the insurance policy issued to Luis G. Morales, husband of herein benefit of widow and children exempt from husband's debt, proceeds of policy
insuring against death by accident insured to widow's benefit free from husband's A.L.R. 1377." (Provident Life & Accident Ins. Co. vs. Rimmer, 12 S. W. 2d Series, 365,
debts. Code 1932, B 8456. American Trust & Banking Co. vs. Lessly, 106 S.W. 2d 551, 367.)
171 Tenn. 511 III A.L.R. 59. For this reason, and because the above-quoted provision of the Rules of Court
Insurance policy, providing for payment in case of accidental death, is "life makes reference to "any life insurance," we are inclined to believe that the
insurance policy" to such extent within state statue prescribing in-contestable exemption there established applies to ordinary life insurance contracts, as well as
period for policies. Code S.C. 1932 ss 7986, 7987. Pacific Mut. Life Ins. Co. of to those which, although intended primarily to indemnify for risks arising from
California vs. Parker, C.C.A.S.C., 71 F. 2d 872, 875. accident, likewise, insure against loss of life due, either to accidental causes, or to
"Life insurance" includes all policies of insurance in which payment of insurance the willful and criminal act of another, which, as such, is not strictly accidental in
money is contingent upon loss of life. . . . Smith vs. Equitable Life Assur. Soc. of U.S., nature. Indeed, it has been held that statutes of this nature seek to enable the head
89 S.W. 2d 165, 167, 169 Tenn. 477. of the family to secure his widow and children from becoming a burden upon the
Insurance policy including a death benefit and a health or accident disability benefit community and, accordingly, should merit a liberal interpretation.
constituted a "life insurance policy" within meaning of laws 1926, c. 118, S. 134, The object of this statue was to enable a husband, when death deprived wife and
imposing privilege tax on insurance companies with different rates as between life children of his support, to secure them from want and to prevent them from
insurance companies and other companies, in view of provisions of Code 1906, ss becoming a charge upon the public. Necessities of the wife and children and the
2576, 2598 (Hemingway's Code 1927, ss 5830, 5856), and Law 1924, c. 191, s I public interest are none the less if the death of the husband be brought about by
(Hemingway's Code 1927, s 5995); it being immaterial that in some policy forms the accident rather than by disease. The intent of the legislature in the enactment of
health and disability feature was more valuable asent a showing that death this statute would not be advanced by the construction of the law upon which the
provision was inserted to avoid the higher tax. Universal Life Ins. Co. vs. State, 121 petitioners insist. (American Trust & Banking Co. vs. Lessly et al., Supreme Court of
So. 849, 850, 155 Miss. 358." (25 Words & Phrases 260, 261, 262.) Tenn., 106 S.W. 2d, 551, 552.)
When the application was made, Harris W. Rimmer carried life insurance with the Under statutes providing to that effect, the proceeds of life insurance are exempt
Equitable Life Assurance Society, for $10,000, payable upon proof of death, with a from the claims of creditors, a limitation being sometimes imposed as to amount,
provision that upon death by accident the amount of insurance payable would be see infra Sec. 40, or as to the beneficiaries entitled to the exemption, see infra
increased to $20,000. The plaintiff insisted that this was life insurance, a disclosure subdivision of this section. Statutes exempting life insurance are regarded as
of which was not called for in question 10, while the defendant insisted it was exemption laws, and not as part of the insurance from law of the state, nor as
accident insurance that should have been disclosed and further insisted that, it designed simply to protect insurer from harassing litigation. Such statutes should be
being a fact material to the risk the failure to disclose the policy in the Equitable Life construed liberally and in the light of, and to give effect to, their purpose of enabling
Assurance Society rendered the policy issued to the applicant void. . . . an individual to provide a fund after his death for his family which will be free from
The court might have gone further and held that the failure of the applicant to the claims of creditors. The exemption privilege is created not by contract but by
characterize the insurance in the Equitable Life Assurance Society as accident legislative grant, and grounds for the exemption of the proceeds of insurance
insurance did not constitute a false answer to the inquiry of what accident or health policies must be found in the statutes. (35 C.J.S. pp. 53-54.)
insurance he was carrying. The policy in the Equitable Life Assurance Society By weight of authority, exemption statutes or rules should be liberally construed
covered loss of life from natural as well as external and accidental causes, and was with a view to giving effect to their beneficent and humane purpose. To this end,
life insurance. The mere addition of the double indemnity clause providing for every reasonable doubt as to whether a given property is or is not exempt should
increased insurance upon proof of death by accident did not divest the policy of its be resolved in favor of exemption. (Comments on the Rules of Court by Moran
character of insurance on life, or make the contract other than life insurance, for [1957 ed.] Vol. 1, p. 564.)
insurance on life includes all policies of insurance in which the payment of the Wherefore, the order appealed from is reversed, and the garnishment in dispute
insurance money is contingent upon the loss of life. Logan vs. Fidelity & Casualty Co., hereby set aside and quashed, with the costs of this instance against plaintiff
146 Mo. 114, 47 S.W. 948. See also Johnson vs. Fidelity & Guaranty Co., 148 Mich. Francisca Gallardo. It is so ordered.
406, 151 N.W. 593, L.R.A. 1916A, 475; Zimmer vs. Central Accidental Co., 207 Pa.
472, 56 A. 1003; Wright vs. Fraternities Health & Accident Ass'n. 107 Me. 418, 78A.
475, 32 L.R.A. (N.S.)461; Metropolitan Life Ins. Co. vs. Ins. Com'r 208 Mass. 386, 94
N.E. 477; Standard Life & Accident Ins. Co. vs. Caroll, 86 F. 567, 41 L.R.A. 194;
Wahl vs. Interstate Business Men's Accident Ass'n 201 Iowa; 1355, 207 N.W. 395, 50
EN BANC The circumstances surrounding the death of Melencio Basilio show that when he
was killed at about seven o'clock in the night of January 25, 1951, he was on duty as
G.R. No. L-8151 December 16, 1955
watchman of the Manila Auto Supply at the corner of Avenida Rizal and Zurbaran;
VIRGINIA CALANOC, petitioner, that it turned out that Atty. Antonio Ojeda who had his residence at the corner of
vs. Zurbaran and Oroquieta, a block away from Basilio's station, had come home that
COURT OF APPEALS and THE PHILIPPINE AMERICAN LIFE INSURANCE night and found that his house was well-lighted, but with the windows closed; that
CO., respondents. getting suspicious that there were culprits in his house, Atty. Ojeda retreated to
look for a policeman and finding Basilio in khaki uniform, asked him to accompany
Lucio Javillonar for petitioner. him to the house with the latter refusing on the ground that he was not a
J. A. Wolfson, Manuel Y. Mecias, Emilio Abello and Anselmo A. Reyes for policeman, but suggesting that Atty. Ojeda should ask the traffic policeman on duty
respondents. at the corner of Rizal Avenue and Zurbaran; that Atty. Ojeda went to the traffic
policeman at said corner and reported the matter, asking the policeman to come
along with him, to which the policeman agreed; that on the way to the Ojeda
residence, the policeman and Atty. Ojeda passed by Basilio and somehow or other
BAUTISTA ANGELO, J.: invited the latter to come along; that as the tree approached the Ojeda residence
and stood in front of the main gate which was covered with galvanized iron, the
This suit involves the collection of P2,000 representing the value of a supplemental fence itself being partly concrete and partly adobe stone, a shot was fired; that
policy covering accidental death which was secured by one Melencio Basilio from immediately after the shot, Atty. Ojeda and the policeman sought cover; that the
the Philippine American Life Insurance Company. The case originated in the policeman, at the request of Atty. Ojeda, left the premises to look for
Municipal Court of Manila and judgment being favorable to the plaintiff it was reinforcement; that it turned out afterwards that the special watchman Melencio
appealed to the court of first instance. The latter court affirmed the judgment but Basilio was hit in the abdomen, the wound causing his instantaneous death; that
on appeal to the Court of Appeals the judgment was reversed and the case is now the shot must have come from inside the yard of Atty. Ojeda, the bullet passing
before us on a petition for review. through a hole waist-high in the galvanized iron gate; that upon inquiry Atty. Ojeda
Melencio Basilio was a watchman of the Manila Auto Supply located at the corner found out that the savings of his children in the amount of P30 in coins kept in his
of Avenida Rizal and Zurbaran. He secured a life insurance policy from the Philippine aparador contained in stockings were taken away, the aparador having been
American Life Insurance Company in the amount of P2,000 to which was attached a ransacked; that a month thereafter the corresponding investigation conducted by
supplementary contract covering death by accident. On January 25, 1951, he died the police authorities led to the arrest and prosecution of four persons in Criminal
of a gunshot wound on the occasion of a robbery committed in the house of Atty. Case No. 15104 of the Court of First Instance of Manila for 'Robbery in an Inhabited
Ojeda at the corner of Oroquieta and Zurbaan streets. Virginia Calanoc, the widow, House and in Band with Murder'.
was paid the sum of P2,000, face value of the policy, but when she demanded the It is contended in behalf of the company that Basilio was killed which "making an
payment of the additional sum of P2,000 representing the value of the arrest as an officer of the law" or as a result of an "assault or murder" committed in
supplemental policy, the company refused alleging, as main defense, that the the place and therefore his death was caused by one of the risks excluded by the
deceased died because he was murdered by a person who took part in the supplementary contract which exempts the company from liability. This contention
commission of the robbery and while making an arrest as an officer of the law was upheld by the Court of Appeals and, in reaching this conclusion, made the
which contingencies were expressly excluded in the contract and have the effect of following comment:
exempting the company from liability.
From the foregoing testimonies, we find that the deceased was a watchman of the
The pertinent facts which need to be considered for the determination of the Manila Auto Supply, and, as such, he was not boud to leave his place and go with
questions raised are those reproduced in the decision of the Court of Appeals as Atty. Ojeda and Policeman Magsanoc to see the trouble, or robbery, that occurred
follows: in the house of Atty. Ojeda. In fact, according to the finding of the lower court, Atty.
Ojeda finding Basilio in uniform asked him to accompany him to his house, but the proof that the death of Basilio is the result of either crime for the record is barren of
latter refused on the ground that he was not a policeman and suggested to Atty. any circumstance showing how the fatal shot was fired. Perhaps this may be
Ojeda to ask help from the traffic policeman on duty at the corner of Rizal Avenue clarified in the criminal case now pending in court as regards the incident but
and Zurbaran, but after Atty. Ojeda secured the help of the traffic policeman, the before that is done anything that might be said on the point would be a mere
deceased went with Ojeda and said traffic policeman to the residence of Ojeda, and conjecture. Nor can it be said that the killing was intentional for there is the
while the deceased was standing in front of the main gate of said residence, he was possibility that the malefactor had fired the shot merely to scare away the people
shot and thus died. The death, therefore, of Basilio, although unexpected, was not around for his own protection and not necessarily to kill or hit the victim. In any
caused by an accident, being a voluntary and intentional act on the part of the one event, while the act may not excempt the triggerman from liability for the damage
wh robbed, or one of those who robbed, the house of Atty. Ojeda. Hence, it is out done, the fact remains that the happening was a pure accident on the part of the
considered opinion that the death of Basilio, though unexpected, cannot be victim. The victim could have been either the policeman or Atty. Ojeda for it cannot
considered accidental, for his death occurred because he left his post and joined be pretended that the malefactor aimed at the deceased precisely because he
policeman Magsanoc and Atty. Ojeda to repair to the latter's residence to see what wanted to take his life.
happened thereat. Certainly, when Basilio joined Patrolman Magsanoc and Atty.
We take note that these defenses are included among the risks exluded in the
Ojeda, he should have realized the danger to which he was exposing himself, yet,
supplementary contract which enumerates the cases which may exempt the
instead of remaining in his place, he went with Atty. Ojeda and Patrolman
company from liability. While as a general rule "the parties may limit the coverage
Magsanoc to see what was the trouble in Atty. Ojeda's house and thus he was
of the policy to certain particular accidents and risks or causes of loss, and may
fatally shot.
expressly except other risks or causes of loss therefrom" (45 C. J. S. 781-782),
We dissent from the above findings of the Court of Appeals. For one thing, Basilio however, it is to be desired that the terms and phraseology of the exception clause
was a watchman of the Manila Auto Supply which was a block away from the house be clearly expressed so as to be within the easy grasp and understanding of the
of Atty. Ojeda where something suspicious was happening which caused the latter insured, for if the terms are doubtful or obscure the same must of necessity be
to ask for help. While at first he declied the invitation of Atty. Ojeda to go with him interpreted or resolved aganst the one who has caused the obscurity. (Article 1377,
to his residence to inquire into what was going on because he was not a regular new Civil Code) And so it has bene generally held that the "terms in an insurance
policeman, he later agreed to come along when prompted by the traffic policeman, policy, which are ambiguous, equivacal, or uncertain . . . are to be construed strictly
and upon approaching the gate of the residence he was shot and died. The and most strongly against the insurer, and liberally in favor of the insured so as to
circumstance that he was a mere watchman and had no duty to heed the call of effect the dominant purpose of indemnity or payment to the insured, especially
Atty. Ojeda should not be taken as a capricious desire on his part to expose his life where a forfeiture is involved" (29 Am. Jur., 181), and the reason for this rule is that
to danger considering the fact that the place he was in duty-bound to guard was he "insured usually has no voice in the selection or arrangement of the words
only a block away. In volunteering to extend help under the situation, he might employed and that the language of the contract is selected with great care and
have thought, rightly or wrongly, that to know the truth was in the interest of his deliberation by experts and legal advisers employed by, and acting exclusively in the
employer it being a matter that affects the security of the neighborhood. No doubt interest of, the insurance company." (44 C. J. S., p. 1174.)
there was some risk coming to him in pursuing that errand, but that risk always
Insurance is, in its nature, complex and difficult for the layman to understand.
existed it being inherent in the position he was holding. He cannot therefore be
Policies are prepared by experts who know and can anticipate the bearings and
blamed solely for doing what he believed was in keeping with his duty as a
possible complications of every contingency. So long as insurance companies insist
watchman and as a citizen. And he cannot be considered as making an arrest as an
upon the use of ambiguous, intricate and technical provisions, which conceal rather
officer of the law, as contended, simply because he went with the traffic policeman,
than frankly disclose, their own intentions, the courts must, in fairness to those who
for certainly he did not go there for that purpose nor was he asked to do so by the
purchase insurance, construe every ambiguity in favor of the insured. (Algoe vs.
policeman.
Pacific Mut. L. Ins. Co., 91 Wash. 324, LRA 1917A, 1237.)lawphi1.net
Much less can it be pretended that Basilio died in the course of an assault or
murder considering the very nature of these crimes. In the first place, there is no
An insurer should not be allowed, by the use of obscure phrases and exceptions, to
defeat the very purpose for which the policy was procured. (Moore vs. Aetna Life
Insurance Co., LRA 1915D, 264.)

We are therefore persuaded to conclude that the circumstances unfolded in the


present case do not warrant the finding that the death of the unfortunate victim
comes within the purview of the exception clause of the supplementary policy and,
hence, do not exempt the company from liability.

Wherefore, reversing the decision appealed from, we hereby order the company to
pay petitioner-appellant the amount of P2,000, with legal interest from January 26,
1951 until fully paid, with costs.
G.R. No. L-25579 March 29, 1972 additional sum of P5,000.00 under the accidental death benefit clause, on the
ground that the insured's death resulted from injuries intentionally inflicted by third
EMILIA T. BIAGTAN, JUAN T. BIAGTAN, JR., MIGUEL T. BIAGTAN, GIL T. BIAGTAN
parties and therefore was not covered. Plaintiffs filed suit to recover, and after due
and GRACIA T. BIAGTAN, plaintiffs-appellees,
hearing the court a quo rendered judgment in their favor. Hence the present appeal
vs.
by the insurer.
THE INSULAR LIFE ASSURANCE COMPANY, LTD., defendant-appellant.
The only issue here is whether under the facts are stipulated and found by the trial
Tanopo, Millora, Serafica, and Sañez for plaintiff-appellees.
court the wounds received by the insured at the hands of the robbers — nine in all,
Araneta, Mendoza and Papa for defendant-appellant. five of them mortal and four non-mortal — were inflicted intentionally. The court,
in ruling negatively on the issue, stated that since the parties presented no evidence
and submitted the case upon stipulation, there was no "proof that the act of
receiving thrust (sic) from the sharp-pointed instrument of the robbers was
MAKALINTAL, J.:p
intended to inflict injuries upon the person of the insured or any other person or
This is an appeal from the decision of the Court of First Instance of Pangasinan in its merely to scare away any person so as to ward off any resistance or obstacle that
Civil Case No. D-1700. might be offered in the pursuit of their main objective which was robbery."

The facts are stipulated. Juan S. Biagtan was insured with defendant InsularLife The trial court committed a plain error in drawing the conclusion it did from the
Assurance Company under Policy No. 398075 for the sum of P5,000.00 and, under a admitted facts. Nine wounds were inflicted upon the deceased, all by means of
supplementary contract denominated "Accidental Death Benefit Clause, for an thrusts with sharp-pointed instruments wielded by the robbers. This is a physical
additional sum of P5,000.00 if "the death of the Insured resulted directly from fact as to which there is no dispute. So is the fact that five of those wounds caused
bodily injury effected solely through external and violent means sustained in an the death of the insured. Whether the robbers had the intent to kill or merely to
accident ... and independently of all other causes." The clause, however,expressly scare the victim or to ward off any defense he might offer, it cannot be denied that
provided that it would not apply where death resulted from an injury"intentionally the act itself of inflicting the injuries was intentional. It should be noted that the
inflicted by another party." exception in the accidental benefit clause invoked by the appellant does not speak
of the purpose — whether homicidal or not — of a third party in causing the
On the night of May 20, 1964, or during the first hours of the following day a band injuries, but only of the fact that such injuries have been "intentionally" inflicted —
of robbers entered the house of the insured Juan S. Biagtan. What happened then is this obviously to distinguish them from injuries which, although received at the
related in the decision of the trial court as follows: hands of a third party, are purely accidental. This construction is the basic idea
...; that on the night of May 20, 1964 or the first hours of May 21, 1964, while the expressed in the coverage of the clause itself, namely, that "the death of the
said life policy and supplementary contract were in full force and effect, the house insured resulted directly from bodily injury effected solely through external and
of insured Juan S. Biagtan was robbed by a band of robbers who were charged in violent means sustained in an accident ... and independently of all other causes." A
and convicted by the Court of First Instance of Pangasinan for robbery with gun which discharges while being cleaned and kills a bystander; a hunter who
homicide; that in committing the robbery, the robbers, on reaching the staircase shoots at his prey and hits a person instead; an athlete in a competitive game
landing on the second floor, rushed towards the door of the second floor room, involving physical effort who collides with an opponent and fatally injures him as a
where they suddenly met a person near the door of oneof the rooms who turned result: these are instances where the infliction of the injury is unintentional and
out to be the insured Juan S. Biagtan who received thrusts from their sharp-pointed therefore would be within the coverage of an accidental death benefit clause such
instruments, causing wounds on the body of said Juan S. Biagtan resulting in his as thatin question in this case. But where a gang of robbers enter a house and
death at about 7 a.m. on the same day, May 21, 1964; coming face to face with the owner, even if unexpectedly, stab him repeatedly, it is
contrary to all reason and logic to say that his injuries are not intentionally inflicted,
Plaintiffs, as beneficiaries of the insured, filed a claim under the policy. The regardless of whether they prove fatal or not. As it was, in the present case they did
insurance company paid the basic amount of P5,000.00 but refused to pay the
prove fatal, and the robbers have been accused and convicted of the crime of In Butero v. Travelers' Acc. Ins. Co., 96 Wis. 536, 65 Am. St. Rep. 61, 71 S.W. 811, the
robbery with homicide. insured was shot three times by a person unknown late on a dark and stormy night,
while working in the coal shed of a railroad company. The policy did not cover
The case of Calanoc vs. Court of Appeals, 98 Phil. 79, is relied upon by the trial court
death resulting from "intentional injuries inflicted by the insured or any other
in support of its decision. The facts in that case, however, are different from those
person." The inquiry was as to the question whether the shooting that caused the
obtaining here. The insured there was a watchman in a certain company, who
insured's death was accidental or intentional; and the Court found that under the
happened to be invited by a policeman to come along as the latter was on his way
facts, showing that the murderer knew his victim and that he fired with intent to
to investigate a reported robbery going on in a private house. As the two of them,
kill, there could be no recovery under the policy which excepted death from
together with the owner of the house, approached and stood in front of the main
intentional injuries inflicted by any person.
gate, a shot was fired and it turned out afterwards that the watchman was hit in the
abdomen, the wound causing his death. Under those circumstances this Court held WHEREFORE, the decision appealed from is reversed and the complaint dismissed,
that it could not be said that the killing was intentional for there was the possibility without pronouncement as to costs.
that the malefactor had fired the shot to scare people around for his own
protection and not necessarrily to kill or hit the victim. A similar possibility is clearly
ruled out by the facts in the case now before Us. For while a single shot fired from a
distance, and by a person who was not even seen aiming at the victim, could indeed
have been fired without intent to kill or injure, nine wounds inflicted with bladed
weapons at close range cannot conceivably be considered as innocent insofar as
such intent is concerned. The manner of execution of the crime permits no other
conclusion.

Court decisions in the American jurisdiction, where similar provisions in accidental


death benefit clauses in insurance policies have been construed, may shed light on
the issue before Us. Thus, it has been held that "intentional" as used in an accident
policy excepting intentional injuries inflicted by the insured or any other person,
etc., implies the exercise of the reasoning faculties, consciousness and
volition.1 Where a provision of the policy excludes intentional injury, it is the
intention of the person inflicting the injury that is controlling.2 If the injuries
suffered by the insured clearly resulted from the intentional act of a third person
the insurer is relieved from liability as stipulated.3

In the case of Hutchcraft's Ex'r v. Travelers' Ins. Co., 87 Ky. 300, 8 S.W. 570, 12 Am.
St. Rep. 484, the insured was waylaid and assassinated for the purpose of robbery.
Two (2) defenses were interposed to the action to recover indemnity, namely: (1)
that the insured having been killed by intentional means, his death was not
accidental, and (2) that the proviso in the policy expressly exempted the insurer
from liability in case the insured died from injuries intentionally inflicted by another
person. In rendering judgment for the insurance company the Court held that while
the assassination of the insured was as to him an unforeseen event and therefore
accidental, "the clause of the proviso that excludes the (insurer's) liability, in case
death or injury is intentionally inflicted by another person, applies to this case."
G.R. No. L-21574 June 30, 1966 while the death of the insured was due to head injury, said injury was sustained
because of his voluntary participation in the contest. It is claimed that the
SIMON DE LA CRUZ, plaintiff and appellee,
participation in the boxing contest was the "means" that produced the injury which,
vs.
in turn, caused the death of the insured. And, since his inclusion in the boxing card
THE CAPITAL INSURANCE and SURETY CO., INC., defendant and appellant.
was voluntary on the part of the insured, he cannot be considered to have met his
Achacoso, Nera and Ocampo for defendant and appellant. death by "accidental means".1äwphï1.ñët
Agustin M. Gramata for plaintiff and appellee.
The terms "accident" and "accidental", as used in insurance contracts, have not
BARRERA, J.: acquired any technical meaning, and are construed by the courts in their ordinary
and common acceptation. Thus, the terms have been taken to mean that which
This is an appeal by the Capital Insurance & Surety Company, Inc., from the decision happen by chance or fortuitously, without intention and design, and which is
of the Court of First Instance of Pangasinan (in Civ Case No. U-265), ordering it to unexpected, unusual, and unforeseen. An accident is an event that takes place
indemnify therein plaintiff Simon de la Cruz for the death of the latter's son, to pay without one's foresight or expectation — an event that proceeds from an unknown
the burial expenses, and attorney's fees. cause, or is an unusual effect of a known cause and, therefore, not expected.1
Eduardo de la Cruz, employed as a mucker in the Itogon-Suyoc Mines, Inc. in Appellant however, would like to make a distinction between "accident or
Baguio, was the holder of an accident insurance policy (No. ITO-BFE-170) accidental" and "accidental means", which is the term used in the insurance policy
underwritten by the Capital Insurance & Surety Co., Inc., for the period beginning involved here. It is argued that to be considered within the protection of the policy,
November 13, 1956 to November 12, 1957. On January 1, 1957, in connection with what is required to be accidental is the means that caused or brought the death and
the celebration of the New Year, the Itogon-Suyoc Mines, Inc. sponsored a boxing not the death itself. It may be mentioned in this connection, that the tendency of
contest for general entertainment wherein the insured Eduardo de la Cruz, a non- court decisions in the United States in recent years is to eliminate the fine
professional boxer participated. In the course of his bout with another person, distinction between the terms "accidental" and "accidental means" and to consider
likewise a non-professional, of the same height, weight, and size, Eduardo slipped them as legally synonymous.2 But, even if we take appellant's theory, the death of
and was hit by his opponent on the left part of the back of the head, causing the insured in the case at bar would still be entitled to indemnification under the
Eduardo to fall, with his head hitting the rope of the ring. He was brought to the policy. The generally accepted rule is that, death or injury does not result from
Baguio General Hospital the following day. The cause of death was reported as accident or accidental means within the terms of an
hemorrhage, intracranial, left. accident-policy if it is the natural result of the insured's voluntary act,
unaccompanied by anything unforeseen except the death or injury.3 There is no
Simon de la Cruz, the father of the insured and who was named beneficiary under
accident when a deliberate act is performed unless some additional, unexpected,
the policy, thereupon filed a claim with the insurance company for payment of the
independent, and unforeseen happening occurs which produces or brings about the
indemnity under the insurance policy. As the claim was denied, De la Cruz instituted
result of injury or death.4 In other words, where the death or injury is not the
the action in the Court of First Instance of Pangasinan for specific performance.
natural or probable result of the insured's voluntary act, or if something unforeseen
Defendant insurer set up the defense that the death of the insured, caused by his
occurs in the doing of the act which produces the injury, the resulting death is
participation in a boxing contest, was not accidental and, therefore, not covered by
within the protection of policies insuring against death or injury from accident.
insurance. After due hearing the court rendered the decision in favor of the plaintiff
which is the subject of the present appeal. In the present case, while the participation of the insured in the boxing contest is
voluntary, the injury was sustained when he slid, giving occasion to the infliction by
It is not disputed that during the ring fight with another non-professional boxer,
his opponent of the blow that threw him to the ropes of the ring. Without this
Eduardo slipped, which was unintentional. At this opportunity, his opponent landed
unfortunate incident, that is, the unintentional slipping of the deceased, perhaps he
on Eduardo's head a blow, which sent the latter to the ropes. That must have
could not have received that blow in the head and would not have died. The fact
caused the cranial injury that led to his death. Eduardo was insured "against death
that boxing is attended with some risks of external injuries does not make any
or disability caused by accidental means". Appellant insurer now contends that
injuries received in the course of the game not accidental. In boxing as in other
equally physically rigorous sports, such as basketball or baseball, death is not
ordinarily anticipated to result. If, therefore, it ever does, the injury or death can
only be accidental or produced by some unforeseen happening or event as what
occurred in this case.

Furthermore, the policy involved herein specifically excluded from its coverage —

(e) Death or disablement consequent upon the Insured engaging in football,


hunting, pigsticking, steeplechasing, polo-playing, racing of any kind,
mountaineering, or motorcycling.

Death or disablement resulting from engagement in boxing contests was not


declared outside of the protection of the insurance contract. Failure of the
defendant insurance company to include death resulting from a boxing match or
other sports among the prohibitive risks leads inevitably to the conclusion that it
did not intend to limit or exempt itself from liability for such death.5

Wherefore, in view of the foregoing considerations, the decision appealed from is


hereby affirmed, with costs against appellant. so ordered
G.R. No. 92383 July 17, 1992 proceeds from an unknown cause, or is an unusual effect of a known case, and
therefore not expected. 4
SUN INSURANCE OFFICE, LTD., petitioner,
vs. An accident is an event which happens without any human agency or, if happening
THE HON. COURT OF APPEALS and NERISSA LIM, respondents. through human agency, an event which, under the circumstances, is unusual to and
not expected by the person to whom it happens. It has also been defined as an
injury which happens by reason of some violence or casualty to the injured without
CRUZ, J.: his design, consent, or voluntary co-operation. 5

The petitioner issued Personal Accident Policy No. 05687 to Felix Lim, Jr. with a face In light of these definitions, the Court is convinced that the incident that resulted in
value of P200,000.00. Two months later, he was dead with a bullet wound in his Lim's death was indeed an accident. The petitioner, invoking the case of De la Cruz
head. As beneficiary, his wife Nerissa Lim sought payment on the policy but her v. Capital Insurance, 6 says that "there is no accident when a deliberate act is
claim was rejected. The petitioner agreed that there was no suicide. It argued, performed unless some additional, unexpected, independent and unforeseen
however that there was no accident either. happening occurs which produces or brings about their injury or death." There was
such a happening. This was the firing of the gun, which was the additional
Pilar Nalagon, Lim's secretary, was the only eyewitness to his death. It happened on unexpected and independent and unforeseen occurrence that led to the insured
October 6, 1982, at about 10 o'clock in the evening, after his mother's birthday person's death.
party. According to Nalagon, Lim was in a happy mood (but not drunk) and was
playing with his handgun, from which he had previously removed the magazine. As The petitioner also cites one of the four exceptions provided for in the insurance
she watched television, he stood in front of her and pointed the gun at her. She contract and contends that the private petitioner's claim is barred by such
pushed it aside and said it might he loaded. He assured her it was not and then provision. It is there stated:
pointed it to his temple. The next moment there was an explosion and Lim slumped
Exceptions —
to the floor. He was dead before he fell. 1
The company shall not be liable in respect of
The widow sued the petitioner in the Regional Trial Court of Zamboanga City and
was sustained. 2 The petitioner was sentenced to pay her P200,000.00, representing 1. Bodily injury
the face value of the policy, with interest at the legal rate; P10,000.00 as moral
xxx xxx xxx
damages; P5,000.00 as exemplary damages; P5,000.00 as actual and compensatory
damages; and P5,000.00 as attorney's fees, plus the costs of the suit. This decision b. consequent upon
was affirmed on appeal, and the motion for reconsideration was denied. 3 The
petitioner then came to this Court to fault the Court of Appeals for approving the i) The insured person attempting to commit suicide or willfully exposing himself to
payment of the claim and the award of damages. needless peril except in an attempt to save human life.

The term "accident" has been defined as follows: To repeat, the parties agree that Lim did not commit suicide. Nevertheless, the
petitioner contends that the insured willfully exposed himself to needless peril and
The words "accident" and "accidental" have never acquired any technical thus removed himself from the coverage of the insurance policy.
signification in law, and when used in an insurance contract are to be construed and
considered according to the ordinary understanding and common usage and speech It should be noted at the outset that suicide and willful exposure to needless peril
of people generally. In-substance, the courts are practically agreed that the words are in pari materia because they both signify a disregard for one's life. The only
"accident" and "accidental" mean that which happens by chance or fortuitously, difference is in degree, as suicide imports a positive act of ending such life whereas
without intention or design, and which is unexpected, unusual, and unforeseen. The the second act indicates a reckless risking of it that is almost suicidal in intent. To
definition that has usually been adopted by the courts is that an accident is an illustrate, a person who walks a tightrope one thousand meters above the ground
event that takes place without one's foresight or expectation — an event that and without any safety device may not actually be intending to commit suicide, but
his act is nonetheless suicidal. He would thus be considered as "willfully exposing relieve the insurer from liability, and none of these exceptions is applicable in the
himself to needless peril" within the meaning of the exception in question. case at bar. **

The petitioner maintains that by the mere act of pointing the gun to hip temple, Lim It bears noting that insurance contracts are as a rule supposed to be interpreted
had willfully exposed himself to needless peril and so came under the exception. liberally in favor of the assured. There is no reason to deviate from this rule,
The theory is that a gun is per se dangerous and should therefore be handled especially in view of the circumstances of this case as above analyzed.
cautiously in every case.
On the second assigned error, however, the Court must rule in favor of the
That posture is arguable. But what is not is that, as the secretary testified, Lim had petitioner. The basic issue raised in this case is, as the petitioner correctly observed,
removed the magazine from the gun and believed it was no longer dangerous. He one of first impression. It is evident that the petitioner was acting in good faith then
expressly assured her that the gun was not loaded. It is submitted that Lim did not it resisted the private respondent's claim on the ground that the death of the
willfully expose himself to needless peril when he pointed the gun to his temple insured was covered by the exception. The issue was indeed debatable and was
because the fact is that he thought it was not unsafe to do so. The act was precisely clearly not raised only for the purpose of evading a legitimate obligation. We hold
intended to assure Nalagon that the gun was indeed harmless. therefore that the award of moral and exemplary damages and of attorney's fees is
unjust and so must be disapproved.
The contrary view is expressed by the petitioner thus:
In order that a person may be made liable to the payment of moral damages, the
Accident insurance policies were never intended to reward the insured for his
law requires that his act be wrongful. The adverse result of an action does not per
tendency to show off or for his miscalculations. They were intended to provide for
se make the act wrongful and subject the act or to the payment of moral damages.
contingencies. Hence, when I miscalculate and jump from the Quezon Bridge into
The law could not have meant to impose a penalty on the right to litigate; such right
the Pasig River in the belief that I can overcome the current, I have wilfully exposed
is so precious that moral damages may not be charged on those who may exercise
myself to peril and must accept the consequences of my act. If I drown I cannot go
it erroneously. For these the law taxes costs. 7
to the insurance company to ask them to compensate me for my failure to swim as
well as I thought I could. The insured in the case at bar deliberately put the gun to The fact that the results of the trial were adverse to Barreto did not alone make his
his head and pulled the trigger. He wilfully exposed himself to peril. act in bringing the action wrongful because in most cases one party will lose; we
would be imposing an unjust condition or limitation on the right to litigate. We hold
The Court certainly agrees that a drowned man cannot go to the insurance
that the award of moral damages in the case at bar is not justified by the facts had
company to ask for compensation. That might frighten the insurance people to
circumstances as well as the law.
death. We also agree that under the circumstances narrated, his beneficiary would
not be able to collect on the insurance policy for it is clear that when he braved the If a party wins, he cannot, as a rule, recover attorney's fees and litigation expenses,
currents below, he deliberately exposed himself to a known peril. since it is not the fact of winning alone that entitles him to recover such damages of
the exceptional circumstances enumerated in Art. 2208. Otherwise, every time a
The private respondent maintains that Lim did not. That is where she says the
defendant wins, automatically the plaintiff must pay attorney's fees thereby putting
analogy fails. The petitioner's hypothetical swimmer knew when he dived off the
a premium on the right to litigate which should not be so. For those expenses, the
Quezon Bridge that the currents below were dangerous. By contrast, Lim did not
law deems the award of costs as sufficient. 8
know that the gun he put to his head was loaded.
WHEREFORE, the challenged decision of the Court of Appeals is AFFIRMED in so far
Lim was unquestionably negligent and that negligence cost him his own life. But it
as it holds the petitioner liable to the private respondent in the sum of P200,000.00
should not prevent his widow from recovering from the insurance policy he
representing the face value of the insurance contract, with interest at the legal rate
obtained precisely against accident. There is nothing in the policy that relieves the
from the date of the filing of the complaint until the full amount is paid, but
insurer of the responsibility to pay the indemnity agreed upon if the insured is
MODIFIED with the deletion of all awards for damages, including attorney's fees,
shown to have contributed to his own accident. Indeed, most accidents are caused
except the costs of the suit.SO ORDERED.
by negligence. There are only four exceptions expressly made in the contract to
II. Insurable interest – 10, 19 2nd part, 25, 184 in relation to 11 4. That during the time the life insurance policy hereinbefore referred to was in
force and effect plaintiff paid from its funds all the insurance premiums due
thereon.
A. In one’s own life, Section 42 of RA 11166
5. That the plaintiff charged as expenses of its business all the said premiums and
A. In one’s spouse and children deducted the same from its gross incomes as reported in its annual income tax
returns, which deductions were allowed by the defendant upon a showing made by
B. Based on dependence for education and support – 195, Family Code the plaintiff that such premiums were legitimate expenses of its (plaintiff's)
business.
C. Based on pecuniary interest
6. That the said A. Velhagen, the insured, had no interest or participation in the
G.R. No. 34774 September 21, 1931
proceeds of said life insurance policy.
EL ORIENTE FABRICA DE TABACOS, INC., plaintiff-appellant,
7. That upon the death of said A. Velhagen in the year 1929, the plaintiff received
vs.
all the proceeds of the said life insurance policy, together with the interests and the
JUAN POSADAS, Collector of Internal Revenue, defendant-appellee.
dividends accruing thereon, aggregating P104,957.88.
Gibbs and McDonough and Roman Ozaeta for appellant.
8. That over the protest of the plaintiff, which claimed exemption under section 4 of
Attorney-General Jaranilla for appellee.
the Income Tax Law, the defendant Collector of Internal Revenue assessed and
MALCOLM, J.: levied the sum of P3,148.74 as income tax on the proceeds of the insurance policy
mentioned in the preceding paragraph, which tax the plaintiff paid under instant
The issue in this case is whether the proceeds of insurance taken by a corporation protest on July 2, 1930; and that defendant overruled said protest on July 9, 1930.
on the life of an important official to indemnify it against loss in case of his death,
are taxable as income under the Philippine Income Tax Law. Thereupon, a decision was handed down which absolved the defendant from the
complaint, with costs against the plaintiff. From this judgment, the plaintiff
The parties submitted the case to the Court of First Instance of Manila for decision appealed, and its counsel now allege that:
upon the following agreed statement of facts:
1. That trial court erred in holding that section 4 of the Income Tax Law (Act No.
1. That the plaintiff is a domestic corporation duly organized and existing under and 2833) is not applicable to the present case.
by virtue of the laws of the Philippine Islands, having its principal office at No. 732
Calle Evangelista, Manila, P.I.; and that the defendant is the duly appointed, 2. The trial court erred in reading into the law certain exceptions and distinctions
qualified and acting Collector of Internal Revenue of the Philippine Islands. not warranted by its clear and unequivocal provisions.

2. That on March 18, 1925, plaintiff, in order to protect itself against the loss that it 3. The trial court erred in assuming that the proceeds of the life insurance policy in
might suffer by reason of the death of its manager, A. Velhagen, who had had more question represented a net profit to the plaintiff when, as a matter of fact, it merely
than thirty-five (35) years of experience in the manufacture of cigars in the represented an indemnity, for the loss suffered by it thru the death of its manager,
Philippine Islands, and whose death would be a serious loss to the plaintiff, the insured.
procured from the Manufacturers Life Insurance Co., of Toronto, Canada, thru its
4. The trial court erred in refusing to hold that the proceeds of the life insurance
local agent E.E. Elser, an insurance policy on the life of the said A. Velhagen for the
policy in question is not taxable income, and in absolving the defendant from the
sum of $50,000, United States currency.
complaint.
3. That the plaintiff, El Oriente, Fabrica de Tabacos, Inc., designated itself as the sole
The Income Tax Law for the Philippines is Act No. 2833, as amended. It is divided
beneficiary of said policy on the life of its said manager.
into four chapters: Chapter I On Individuals, Chapter II On Corporations, Chapter III
General Administrative Provisions, and Chapter IV General Provisions. In chapter I signifies that when the plaintiff received P104,957.88 from the insurance on the life
On Individuals, is to be found section 4 which provides that, "The following incomes of its manager, it thereby realized a net profit in this amount. It is true that the
shall be exempt from the provisions of this law: (a) The proceeds of life insurance Income Tax Law, in exempting individual beneficiaries, speaks of the proceeds of
policies paid to beneficiaries upon the death of the insured ... ." Section 10, as life insurance policies as income, but this is a very slight indication of legislative
amended, in Chapter II On Corporations, provides that, There shall be levied, intention. In reality, what the plaintiff received was in the nature of an indemnity
assessed, collected, and paid annually upon the total net income received in the for the loss which it actually suffered because of the death of its manager.
preceding calendar year from all sources by every corporation ... a tax of three per
To quote the exact words in the cited case of Chief Justice Taft delivering the
centum upon such income ... ." Section 11 in the same chapter, provides the
opinion of the court:
exemptions under the law, but neither here nor in any other section is reference
made to the provisions of section 4 in Chapter I. It is earnestly pressed upon us that proceeds of life insurance paid on the death of
the insured are in fact capital, and cannot be taxed as income under the Sixteenth
Under the view we take of the case, it is sufficient for our purposes to direct
Amendment. Eisner vs. Macomber, 252 U.S., 189, 207; Merchants' Loan & Trust
attention to the anomalous and vague condition of the law. It is certain that the
Co. vs. Smietanka, 255 U.S., 509, 518. We are not required to meet this question. It
proceeds of life insurance policies are exempt. It is not so certain that the proceeds
is enough to sustain our construction of the act to say that proceeds of a life
of life insurance policies paid to corporate beneficiaries upon the death of the
insurance policy paid on the death of the insured are not usually classed as income.
insured are likewise exempt. But at least, it may be said that the law is indefinite in
phraseology and does not permit us unequivocally to hold that the proceeds of life . . . Life insurance in such a case is like that of fire and marine insurance, — a
insurance policies received by corporations constitute income which is taxable. contract of indemnity. Central Nat. Bank vs. Hume, 128 U.S., 195. The benefit to be
gained by death has no periodicity. It is a substitution of money value for something
The situation will be better elucidated by a brief reference to laws on the same
permanently lost, either in a house, a ship, or a life. Assuming, without deciding,
subject in the United States. The Income Tax Law of 1916 extended to the
that Congress could call the proceeds of such indemnity income, and validly tax it as
Philippine Legislature, when it came to enact Act No. 2833, to copy the American
such, we think that, in view of the popular conception of the life insurance as
statute. Subsequently, the Congress of the United States enacted its Income Tax
resulting in a single addition of a total sum to the resources of the beneficiary, and
Law of 1919, in which certain doubtful subjects were clarified. Thus, as to the point
not in a periodical return, such a purpose on its part should be express, as it
before us, it was made clear, when not only in the part of the law concerning
certainly is not here.
individuals were exemptions provided for beneficiaries, but also in the part
concerning corporations, specific reference was made to the exemptions in favor of Considering, therefore, the purport of the stipulated facts, considering the
individuals, thereby making the same applicable to corporations. This was uncertainty of Philippine law, and considering the lack of express legislative
authoritatively pointed out and decided by the United States Supreme Court in the intention to tax the proceeds of life insurance policies paid to corporate
case of United States vs. Supplee-Biddle Hardware Co. ( [1924], 265 U.S., 189), beneficiaries, particularly when in the exemption in favor of individual beneficiaries
which involved facts quite similar to those before us. We do not think the decision in the chapter on this subject, the clause is inserted "exempt from the provisions of
of the higher court in this case is necessarily controlling on account of the this law," we deem it reasonable to hold the proceeds of the life insurance policy in
divergences noted in the federal statute and the local statute, but we find in the question as representing an indemnity and not taxable income.
decision certain language of a general nature which appears to furnish the clue to
the correct disposition of the instant appeal. Conceding, therefore, without The foregoing pronouncement will result in the judgment being reversed and in
necessarily having to decide, the assignments of error Nos. 1 and 2 are not well another judgment being rendered in favor of the plaintiff and against the defendant
taken, we would turn to the third assignment of error. for the sum of P3,148.74. So ordered, without costs in either instance.

It will be recalled that El Oriente, Fabrica de Tabacos, Inc., took out the insurance on
the life of its manager, who had had more than thirty-five years' experience in the
manufacture of cigars in the Philippines, to protect itself against the loss it might
suffer by reason of the death of its manager. We do not believe that this fact
E. Based on legal obligation I. As to the first assignment of error, little need be said. The insurance contract,
Exhibit A, is printed in the English common form of marine policies. One of the
III. The Life Insurance Policy – 49-52, 56, 232-237
clauses of the document originally read as follows:

Touching the Adventures and Perils which the said National Union Fire Insurance
G.R. No. L-32986 November 11, 1930 Company is content to bear, and to take upon them in this Voyage; they are of the
Seas, Men-of-War, Fire, Pirates, Rovers, Thieves, Jettison, Letters of Mart and
FRANCISCO JARQUE, plaintiff-appellee, Countermart, Surprisals, and Takings at Sea. Arrest, Restraint and Detainments, of
vs. all Kings Princes and People of what Nation, Condition or Quality so ever; Barratry
SMITH, BELL & CO., LTD., ET AL., defendants. of the Master and Marines, and of all other Perils, Losses and Misfortunes, that
UNION FIRE INSURANCE CO., appellant. have or shall come to the Hurt, Detriment, or Damage of the said Vessel or any part
thereof; and in case of any Loss or Misfortunes, it shall be lawful for the Assured, his
Benj. S. Ohnick for appellant.
or their Factors, Servants, or assigns, to sue, labour and travel for, in and about the
Vicente Pelaez for appellee.
Defense. Safeguard, and recovery of the said Vessel or any Charges whereof the
said Company, will contribute, according to the rate and quantity of the sum herein
assured shall be of as much force and Virtue as the surest Writing or Policy of
Insurance made in LONDON.
OSTRAND, J.:
Attached to the policy over and above the said clause is a "rider" containing
The plaintiff was the owner of the motorboat Pandan and held a marine insurance typewritten provisions, among which appears in capitalized type the following
policy for the sum of P45,000 on the boat, the policy being issued by the National clause:
Union Fire Insurance Company and according to the provisions of a "rider" attached
to the policy, the insurance was against the "absolute total loss of the vessel only." AGAINST THE ABSOLUTE TOTAL LOSS OF THE VESSEL ONLY, AND TO PAY
On October 31, 1928, the ship ran into very heavy sea off the Islands of Ticlin, and it PROPORTIONATE SALVAGE CHARGES OF TEH DECLARED VALUE.
became necessary to jettison a portion of the cargo. As a result of the jettison, the
At the bottom of the same rider following the type written provisions therein set
National Union Fire Insurance Company was assessed in the sum of P2,610.86 as its
forth are the following words: "Attaching to and forming part of the National Union
contribution to the general average. The insurance company, insisting that its
Fire Insurance Co., Hull Policy No. 1055."
obligation did not extend beyond the insurance of the "absolute total loss of the
vessel only, and to pay proportionate salvage of the declared value," refused to It is a well settled rule that in case repugnance exists between written and printed
contribute to the settlement of the general average. The present action was portions of a policy, the written portion prevails, and there can be no question that
thereupon instituted, and after trial the court below rendered judgment in favor of as far as any inconsistency exists, the above-mentioned typed "rider" prevails over
the plaintiff and ordered the defendant National Union Fire Insurance Company to the printed clause it covers. Section 291 of the Code of Civil Procedure provides that
pay the plaintiff the sum of P2,610.86 as its part of the indemnity for the general "when an instrument consists partly of written words and partly of a printed form
average brought about by the jettison of cargo. The insurance company appealed to and the two are inconsistent, the former controls the latter." (See also Joyce on
this court and assigns as errors (1) "that the lower court erred in disregarding the Insurance, 2d ed., sec. 224, page 600; Arnould on Marine Insurance, 9th ed., sec.
typewritten clause endorsed upon the policy, Exhibit A, expressly limiting insurer's 73; Marine Equipment Corporation vs. Automobile Insurance Co., 24 Fed. (2d), 600;
liability thereunder of the total loss of the wooden vessel Pandan and to and Marine Insurance Company vs. McLahanan, 290 Fed., 685, 688.)
proportionate salvage charges," and (2) "that the lower court erred in concluding
that defendant and appellant, National Union Fire Insurance Company is liable to II. In the absence of positive legislation to the contrary, the liability of the defendant
contribute to the general average resulting from the jettison of a part of said insurance company on its policy would, perhaps, be limited to "absolute loss of the
vessel's cargo." vessel only, and to pay proportionate salvage of the declared value." But the policy
was executed in this jurisdiction and "warranted to trade within the waters of the
Philippine Archipelago only." Here the liability for contribution in general average is
not based on the express terms of the policy, but rest upon the theory that from
the relation of the parties and for their benefit, a quasi contract is implied by law.
Article 859 of the Code of Commerce is still in force and reads as follows:

ART. 859. The underwriters of the vessel, of the freight, and of the cargo shall be
obliged to pay for the indemnity of the gross average in so far as is required of each
one of these objects respectively.

The article is mandatory in its terms, and the insurers, whether for the vessel or for
the freight or for the cargo, are bound to contribute to the indemnity of the general
average. And there is nothing unfair in that provisions; it simply places the insurer
on the same footing as other persons who have an interest in the vessel, or the
cargo therein at the time of the occurrence of the general average and who are
compelled to contribute (art. 812, Code of Commerce).

In the present case it is not disputed that the ship was in grave peril and that the
jettison of part of the cargo was necessary. If the cargo was in peril to the extent of
call for general average, the ship must also have been in great danger, possibly
sufficient to cause its absolute loss. The jettison was therefore as much to the
benefit of the underwriter as to the owner of the cargo. The latter was compelled
to contribute to the indemnity; why should not the insurer be required to do
likewise? If no jettison had take place and if the ship by reason thereof had
foundered, the underwriter's loss would have been many times as large as the
contribution now demanded. lawphil.net

The appealed judgment is affirmed with the cost against the appellant. So ordered.
G.R. No. 115278 May 23, 1995 4. The Security Guard Atiga was assigned by Unicorn Security Services, Inc. with the
plaintiff by virtue of a contract of Security Service executed on October 25, 1982, a
FORTUNE INSURANCE AND SURETY CO., INC., petitioner,
duplicate original copy of which is hereto attached as Exhibit "C";
vs.
COURT OF APPEALS and PRODUCERS BANK OF THE PHILIPPINES, respondents. 5. After an investigation conducted by the Pasay police authorities, the driver
Magalong and guard Atiga were charged, together with Edelmer Bantigue Y Eulalio,
Reynaldo Aquino and John Doe, with violation of P.D. 532 (Anti-Highway Robbery
DAVIDE, JR., J.: Law) before the Fiscal of Pasay City. A copy of the complaint is hereto attached as
Exhibit "D";
The fundamental legal issue raised in this petition for review on certiorari is
whether the petitioner is liable under the Money, Security, and Payroll Robbery 6. The Fiscal of Pasay City then filed an information charging the aforesaid persons
policy it issued to the private respondent or whether recovery thereunder is with the said crime before Branch 112 of the Regional Trial Court of Pasay City. A
precluded under the general exceptions clause thereof. Both the trial court and the copy of the said information is hereto attached as Exhibit "E." The case is still being
Court of Appeals held that there should be recovery. The petitioner contends tried as of this date;
otherwise.
7. Demands were made by the plaintiff upon the defendant to pay the amount of
This case began with the filing with the Regional Trial Court (RTC) of Makati, Metro the loss of P725,000.00, but the latter refused to pay as the loss is excluded from
Manila, by private respondent Producers Bank of the Philippines (hereinafter the coverage of the insurance policy, attached hereto as Exhibit "A," specifically
Producers) against petitioner Fortune Insurance and Surety Co., Inc. (hereinafter under page 1 thereof, "General Exceptions" Section (b), which is marked as Exhibit
Fortune) of a complaint for recovery of the sum of P725,000.00 under the policy "A-1," and which reads as follows:
issued by Fortune. The sum was allegedly lost during a robbery of Producer's
GENERAL EXCEPTIONS
armored vehicle while it was in transit to transfer the money from its Pasay City
Branch to its head office in Makati. The case was docketed as Civil Case No. 1817 The company shall not be liable under this policy in report of
and assigned to Branch 146 thereof.
xxx xxx xxx
After joinder of issues, the parties asked the trial court to render judgment based
(b) any loss caused by any dishonest, fraudulent or criminal act of the insured or
on the following stipulation of facts:
any officer, employee, partner, director, trustee or authorized representative of the
1. The plaintiff was insured by the defendants and an insurance policy was issued, Insured whether acting alone or in conjunction with others. . . .
the duplicate original of which is hereto attached as Exhibit "A";
8. The plaintiff opposes the contention of the defendant and contends that Atiga
2. An armored car of the plaintiff, while in the process of transferring cash in the and Magalong are not its "officer, employee, . . . trustee or authorized
sum of P725,000.00 under the custody of its teller, Maribeth Alampay, from its representative . . . at the time of the robbery.1
Pasay Branch to its Head Office at 8737 Paseo de Roxas, Makati, Metro Manila on
On 26 April 1990, the trial court rendered its decision in favor of Producers. The
June 29, 1987, was robbed of the said cash. The robbery took place while the
dispositive portion thereof reads as follows:
armored car was traveling along Taft Avenue in Pasay City;
WHEREFORE, premises considered, the Court finds for plaintiff and against
3. The said armored car was driven by Benjamin Magalong Y de Vera, escorted by
defendant, and
Security Guard Saturnino Atiga Y Rosete. Driver Magalong was assigned by PRC
Management Systems with the plaintiff by virtue of an Agreement executed on (a) orders defendant to pay plaintiff the net amount of P540,000.00 as liability
August 7, 1983, a duplicate original copy of which is hereto attached as Exhibit "B"; under Policy No. 0207 (as mitigated by the P40,000.00 special clause deduction and
by the recovered sum of P145,000.00), with interest thereon at the legal rate, until
fully paid;
(b) orders defendant to pay plaintiff the sum of P30,000.00 as and for attorney's Appeals, 207 SCRA 669; Sun Insurance Office, Ltd. vs. Court of Appeals, 211 SCRA
fees; and 554). Contracts of insurance, like other contracts, are to be construed according to
the sense and meaning of the terms which the parties themselves have used. If
(c) orders defendant to pay costs of suit.
such terms are clear and unambiguous, they must be taken and understood in their
All other claims and counterclaims are accordingly dismissed forthwith. plain, ordinary and popular sense (New Life Enterprises Case, supra, p. 676; Sun
Insurance Office, Ltd. vs. Court of Appeals, 195 SCRA 193).
SO ORDERED. 2
The language used by defendant-appellant in the above quoted stipulation is plain,
The trial court ruled that Magalong and Atiga were not employees or ordinary and simple. No other interpretation is necessary. The word "employee"
representatives of Producers. It Said: must be taken to mean in the ordinary sense.
The Court is satisfied that plaintiff may not be said to have selected and engaged The Labor Code is a special law specifically dealing with/and specifically designed to
Magalong and Atiga, their services as armored car driver and as security guard protect labor and therefore its definition as to employer-employee relationships
having been merely offered by PRC Management and by Unicorn Security and insofar as the application/enforcement of said Code is concerned must necessarily
which latter firms assigned them to plaintiff. The wages and salaries of both be inapplicable to an insurance contract which defendant-appellant itself had
Magalong and Atiga are presumably paid by their respective firms, which alone formulated. Had it intended to apply the Labor Code in defining what the word
wields the power to dismiss them. Magalong and Atiga are assigned to plaintiff in "employee" refers to, it must/should have so stated expressly in the insurance
fulfillment of agreements to provide driving services and property protection as policy.
such — in a context which does not impress the Court as translating into plaintiff's
power to control the conduct of any assigned driver or security guard, beyond Said driver and security guard cannot be considered as employees of plaintiff-
perhaps entitling plaintiff to request are replacement for such driver guard. The appellee bank because it has no power to hire or to dismiss said driver and security
finding is accordingly compelled that neither Magalong nor Atiga were plaintiff's guard under the contracts (Exhs. 8 and C) except only to ask for their replacements
"employees" in avoidance of defendant's liability under the policy, particularly the from the contractors.5
general exceptions therein embodied.
On 20 June 1994, Fortune filed this petition for review on certiorari. It alleges that
Neither is the Court prepared to accept the proposition that driver Magalong and the trial court and the Court of Appeals erred in holding it liable under the
guard Atiga were the "authorized representatives" of plaintiff. They were merely an insurance policy because the loss falls within the general exceptions clause
assigned armored car driver and security guard, respectively, for the June 29, 1987 considering that driver Magalong and security guard Atiga were Producers'
money transfer from plaintiff's Pasay Branch to its Makati Head Office. Quite plainly authorized representatives or employees in the transfer of the money and payroll
— it was teller Maribeth Alampay who had "custody" of the P725,000.00 cash being from its branch office in Pasay City to its head office in Makati.
transferred along a specified money route, and hence plaintiff's then designated
According to Fortune, when Producers commissioned a guard and a driver to
"messenger" adverted to in the policy. 3
transfer its funds from one branch to another, they effectively and necessarily
Fortune appealed this decision to the Court of Appeals which docketed the case as became its authorized representatives in the care and custody of the money.
CA-G.R. CV No. 32946. In its decision 4 promulgated on 3 May 1994, it affirmed in Assuming that they could not be considered authorized representatives, they were,
toto the appealed decision. nevertheless, employees of Producers. It asserts that the existence of an employer-
employee relationship "is determined by law and being such, it cannot be the
The Court of Appeals agreed with the conclusion of the trial court that Magalong subject of agreement." Thus, if there was in reality an employer-employee
and Atiga were neither employees nor authorized representatives of Producers and relationship between Producers, on the one hand, and Magalong and Atiga, on the
ratiocinated as follows: other, the provisions in the contracts of Producers with PRC Management System
for Magalong and with Unicorn Security Services for Atiga which state that
A policy or contract of insurance is to be construed liberally in favor of the insured
Producers is not their employer and that it is absolved from any liability as an
and strictly against the insurance company (New Life Enterprises vs. Court of
employer, would not obliterate the relationship.
Fortune points out that an employer-employee relationship depends upon four responsible for his faithful discharge of his duties and responsibilities, and since
standards: (1) the manner of selection and engagement of the putative employee; Producers paid the monthly compensation of P1,400.00 per driver to PRC
(2) the mode of payment of wages; (3) the presence or absence of a power to Management Systems and not to Magalong, it is clear that Magalong was not
dismiss; and (4) the presence and absence of a power to control the putative Producers' employee. As to Atiga, Producers relies on the provision of its contract
employee's conduct. Of the four, the right-of-control test has been held to be the with Unicorn Security Services which provides that the guards of the latter "are in
decisive factor. 6 It asserts that the power of control over Magalong and Atiga was no sense employees of the CLIENT."
vested in and exercised by Producers. Fortune further insists that PRC Management
There is merit in this petition.
System and Unicorn Security Services are but "labor-only" contractors under Article
106 of the Labor Code which provides: It should be noted that the insurance policy entered into by the parties is a theft or
robbery insurance policy which is a form of casualty insurance. Section 174 of the
Art. 106. Contractor or subcontractor. — There is "labor-only" contracting where
Insurance Code provides:
the person supplying workers to an employer does not have substantial capital or
investment in the form of tools, equipment, machineries, work premises, among Sec. 174. Casualty insurance is insurance covering loss or liability arising from
others, and the workers recruited and placed by such persons are performing accident or mishap, excluding certain types of loss which by law or custom are
activities which are directly related to the principal business of such employer. In considered as falling exclusively within the scope of insurance such as fire or
such cases, the person or intermediary shall be considered merely as an agent of marine. It includes, but is not limited to, employer's liability insurance, public
the employer who shall be responsible to the workers in the same manner and liability insurance, motor vehicle liability insurance, plate glass insurance, burglary
extent as if the latter were directly employed by him. and theft insurance, personal accident and health insurance as written by non-life
insurance companies, and other substantially similar kinds of insurance. (emphases
Fortune thus contends that Magalong and Atiga were employees of Producers,
supplied)
following the ruling in International Timber Corp. vs. NLRC 7 that a finding that a
contractor is a "labor-only" contractor is equivalent to a finding that there is an Except with respect to compulsory motor vehicle liability insurance, the Insurance
employer-employee relationship between the owner of the project and the Code contains no other provisions applicable to casualty insurance or to robbery
employees of the "labor-only" contractor. insurance in particular. These contracts are, therefore, governed by the general
provisions applicable to all types of insurance. Outside of these, the rights and
On the other hand, Producers contends that Magalong and Atiga were not its
obligations of the parties must be determined by the terms of their contract, taking
employees since it had nothing to do with their selection and engagement, the
into consideration its purpose and always in accordance with the general principles
payment of their wages, their dismissal, and the control of their conduct. Producers
of insurance law. 9
argued that the rule in International Timber Corp. is not applicable to all cases but
only when it becomes necessary to prevent any violation or circumvention of the It has been aptly observed that in burglary, robbery, and theft insurance, "the
Labor Code, a social legislation whose provisions may set aside contracts entered opportunity to defraud the insurer — the moral hazard — is so great that insurers
into by parties in order to give protection to the working man. have found it necessary to fill up their policies with countless restrictions, many
designed to reduce this hazard. Seldom does the insurer assume the risk of all
Producers further asseverates that what should be applied is the rule in American
losses due to the hazards insured against." 10 Persons frequently excluded under
President Lines vs. Clave, 8 to wit:
such provisions are those in the insured's service and employment. 11 The purpose
In determining the existence of employer-employee relationship, the following of the exception is to guard against liability should the theft be committed by one
elements are generally considered, namely: (1) the selection and engagement of having unrestricted access to the property. 12 In such cases, the terms specifying the
the employee; (2) the payment of wages; (3) the power of dismissal; and (4) the excluded classes are to be given their meaning as understood in common
power to control the employee's conduct. speech. 13 The terms "service" and "employment" are generally associated with the
idea of selection, control, and compensation. 14
Since under Producers' contract with PRC Management Systems it is the latter
which assigned Magalong as the driver of Producers' armored car and was
A contract of insurance is a contract of adhesion, thus any ambiguity therein should employees under a "labor-only" contract as employees of the party employing them
be resolved against the insurer, 15 or it should be construed liberally in favor of the and not of the party who supplied them to the employer. 22
insured and strictly against the insurer. 16 Limitations of liability should be regarded
Fortune claims that Producers' contracts with PRC Management Systems and
with extreme jealousy and must be construed
Unicorn Security Services are "labor-only" contracts.
in such a way, as to preclude the insurer from non-compliance with its
obligation. 17 It goes without saying then that if the terms of the contract are clear Producers, however, insists that by the express terms thereof, it is not the employer
and unambiguous, there is no room for construction and such terms cannot be of Magalong. Notwithstanding such express assumption of PRC Management
enlarged or diminished by judicial construction. 18 Systems and Unicorn Security Services that the drivers and the security guards each
shall supply to Producers are not the latter's employees, it may, in fact, be that it is
An insurance contract is a contract of indemnity upon the terms and conditions
because the contracts are, indeed, "labor-only" contracts. Whether they are is, in
specified therein. 19 It is settled that the terms of the policy constitute the measure
the light of the criteria provided for in Article 106 of the Labor Code, a question of
of the insurer's liability. 20 In the absence of statutory prohibition to the contrary,
fact. Since the parties opted to submit the case for judgment on the basis of their
insurance companies have the same rights as individuals to limit their liability and
stipulation of facts which are strictly limited to the insurance policy, the contracts
to impose whatever conditions they deem best upon their obligations not
with PRC Management Systems and Unicorn Security Services, the complaint for
inconsistent with public policy.
violation of P.D. No. 532, and the information therefor filed by the City Fiscal of
With the foregoing principles in mind, it may now be asked whether Magalong and Pasay City, there is a paucity of evidence as to whether the contracts between
Atiga qualify as employees or authorized representatives of Producers under Producers and PRC Management Systems and Unicorn Security Services are "labor-
paragraph (b) of the general exceptions clause of the policy which, for easy only" contracts.
reference, is again quoted:
But even granting for the sake of argument that these contracts were not "labor-
GENERAL EXCEPTIONS only" contracts, and PRC Management Systems and Unicorn Security Services were
truly independent contractors, we are satisfied that Magalong and Atiga were, in
The company shall not be liable under this policy in respect of
respect of the transfer of Producer's money from its Pasay City branch to its head
xxx xxx xxx office in Makati, its "authorized representatives" who served as such with its teller
Maribeth Alampay. Howsoever viewed, Producers entrusted the three with the
(b) any loss caused by any dishonest, fraudulent or criminal act of the insured or specific duty to safely transfer the money to its head office, with Alampay to be
any officer, employee, partner, director, trustee or authorized representative of the responsible for its custody in transit; Magalong to drive the armored vehicle which
Insured whether acting alone or in conjunction with others. . . . (emphases would carry the money; and Atiga to provide the needed security for the money,
supplied) the vehicle, and his two other companions. In short, for these particular tasks, the
three acted as agents of Producers. A "representative" is defined as one who
There is marked disagreement between the parties on the correct meaning of the
represents or stands in the place of another; one who represents others or another
terms "employee" and "authorized representatives."
in a special capacity, as an agent, and is interchangeable with "agent." 23
It is clear to us that insofar as Fortune is concerned, it was its intention to exclude
In view of the foregoing, Fortune is exempt from liability under the general
and exempt from protection and coverage losses arising from dishonest, fraudulent,
exceptions clause of the insurance policy.
or criminal acts of persons granted or having unrestricted access to Producers'
money or payroll. When it used then the term "employee," it must have had in WHEREFORE , the instant petition is hereby GRANTED. The decision of the Court of
mind any person who qualifies as such as generally and universally understood, or Appeals in CA-G.R. CV No. 32946 dated 3 May 1994 as well as that of Branch 146 of
jurisprudentially established in the light of the four standards in the determination the Regional Trial Court of Makati in Civil Case No. 1817 are REVERSED and SET
of the employer-employee relationship, 21 or as statutorily declared even in a ASIDE. The complaint in Civil Case No. 1817 is DISMISSED.
limited sense as in the case of Article 106 of the Labor Code which considers the
G.R. No. L-15774 November 29, 1920 null and void ab initio, and the Company shall be held not to have been on the risk
at all, but in such case the amount herein acknowledged shall be returned.
PILAR C. DE LIM, plaintiff-appellant,
vs. [SEAL.] (Sgd.) T. B. MACAULAY, President.
SUN LIFE ASSURANCE COMPANY OF CANADA, defendant-appellee. (Sgd.) A. F. Peters, Agent.

Sanz and Luzuriaga for appellant. Our duty in this case is to ascertain the correct meaning of the document above
Cohn and Fisher for appellee. quoted. A perusal of the same many times by the writer and by other members of
the court leaves a decided impression of vagueness in the mind. Apparently it is to
be a provisional policy "for four months only from the date of this application." We
use the term "apparently" advisedly, because immediately following the words
MALCOLM, J.: fixing the four months period comes the word "provided" which has the meaning of
"if." Otherwise stated, the policy for four months is expressly made subjected to the
This is an appeal by plaintiff from an order of the Court of First Instance of affirmative condition that "the company shall confirm this agreement by issuing a
Zamboanga sustaining a demurrer to plaintiff's complaint upon the ground that it policy on said application when the same shall be submitted to the head office in
fails to state a cause of action. Montreal." To reenforce the same there follows the negative condition —
As the demurrer had the effect of admitting the material facts set forth in the Should the company not issue such a policy, then this agreement shall be null and
complaint, the facts are those alleged by the plaintiff. On July 6, 1917, Luis Lim y void ab initio, and the company shall be held not to have been on the risk."
Garcia of Zamboanga made application to the Sun Life Assurance Company of Certainly, language could hardly be used which would more clearly stipulate that
Canada for a policy of insurance on his life in the sum of P5,000. In his application the agreement should not go into effect until the home office of the company
Lim designated his wife, Pilar C. de Lim, the plaintiff herein, as the beneficiary. The should confirm it by issuing a policy. As we read and understand the so-called
first premium of P433 was paid by Lim, and upon such payment the company issued provisional policy it amounts to nothing but an acknowledgment on behalf of the
what was called a "provisional policy." Luis Lim y Garcia died on August 23, 1917, company, that it has received from the person named therein the sum of money
after the issuance of the provisional policy but before approval of the application by agreed upon as the first year's premium upon a policy to be issued upon the
the home office of the insurance company. The instant action is brought by the application, if the application is accepted by the company.
beneficiary, Pilar C. de Lim, to recover from the Sun Life Assurance Company of
Canada the sum of P5,000, the amount named in the provisional policy. It is of course a primary rule that a contract of insurance, like other contracts, must
be assented to by both parties either in person or by their agents. So long as an
The "provisional policy" upon which this action rests reads as follows: application for insurance has not been either accepted or rejected, it is merely an
offer or proposal to make a contract. The contract, to be binding from the date of
Received (subject to the following stipulations and agreements) the sum of four
the application, must have been a completed contract, one that leaves nothing to
hundred and thirty-three pesos, being the amount of the first year's premium for a
be done, nothing to be completed, nothing to be passed upon, or determined,
Life Assurance Policy on the life of Mr. Luis D. Lim y Garcia of Zamboanga for
before it shall take effect. There can be no contract of insurance unless the minds of
P5,000, for which an application dated the 6th day of July, 1917, has been made to
the parties have met in agreement. Our view is, that a contract of insurance was not
the Sun Life Assurance Company of Canada.
here consummated by the parties.lawph!l.net
The above-mentioned life is to be assured in accordance with the terms and
Appellant relies on Joyce on Insurance. Beginning at page 253, of Volume I, Joyce
conditions contained or inserted by the Company in the policy which may be
states the general rule concerning the agent's receipt pending approval or issuance
granted by it in this particular case for four months only from the date of the
of policy. The first rule which Joyce lays down is this: If the act of acceptance of the
application, provided that the Company shall confirm this agreement by issuing a
risk by the agent and the giving by him of a receipt, is within the scope of the
policy on said application when the same shall be submitted to the Head Office in
agent's authority, and nothing remains but to issue a policy, then the receipt will
Montreal. Should the Company not issue such a policy, then this agreement shall be
bind the company. This rule does not apply, for while here nothing remained but to
issue the policy, this was made an express condition to the contract. The second entered into in this instance, but we do not think so. On the contrary, the clause in
rule laid down by Joyce is this: Where an agreement is made between the applicant the application and the receipt given by the solicitor, which are to be read together,
and the agent whether by signing an application containing such condition, or stipulate expressly that the insurance shall become effective only when the
otherwise, that no liability shall attach until the principal approves the risk and a "application shall be approved and the policy duly signed by the secretary at the
receipt is given buy the agent, such acceptance is merely conditional, and it head office of the company and issued." It constituted no agreement at all for
subordinated to the act of the company in approving or rejecting; so in life preliminary or temporary insurance; Mohrstadt vs. Mutual Life Ins. Co., 115 Fed.,
insurance a "binding slip" or "binding receipt" does not insure of itself. This is the 81, 52 C. C. A., 675; Steinle vs. New York Life Ins. Co., 81 Fed., 489, 26 C. C. A., 491."
rule which we believe applies to the instant case. The third rule announced by Joyce (See further Weinfeld vs. Mutual Reserve Fund Life Ass'n. [1892], 53 Fed, 208'
is this: Where the acceptance by the agent is within the scope of his authority a Mohrstadt vs. Mutual Life Insurance Co. [1902], 115 Fed., 81; Insurance co. vs.
receipt containing a contract for insurance for a specific time which is not absolute Young's Administrator [1875], 90 U. S., 85; Chamberlain vs. Prudential Insurance
but conditional, upon acceptance or rejection by the principal, covers the specified Company of America [1901], 109 Wis., 4; Shawnee Mut. Fire Ins. Co. vs. McClure
period unless the risk is declined within that period. The case cited by Joyce to [1913], 39 Okla., 509; Dorman vs. Connecticut Fire Ins. Co. [1914], 51 contra, Starr
substantiate the last principle is that a Goodfellow vs. Times & Beacon Assurance vs. Mutual Life Ins. Co. [1905], 41 Wash., 228.)
Com. (17 U. C. Q. B., 411), not available.
We are of the opinion that the trial court committed no error in sustaining the
The two cases most nearly in point come from the federal courts and the Supreme demurrer and dismissing the case. It is to be noted, however, that counsel for
Court of Arkansas. appellee admits the liability of the company for the return of the first premium to
the estate of the deceased. It is not to be doubted but that the Sun Life Assurance
In the case of Steinle vs. New York Life Insurance Co. ([1897], 81 Fed., 489} the facts
Company of Canada will immediately, on the promulgation of this decision, pay to
were that the amount of the first premium had been paid to an insurance agent and
the estate of the late Luis Lim y Garcia the of P433.
a receipt given therefor. The receipt, however, expressly declared that if the
application was accepted by the company, the insurance shall take effect from the The order appealed from, in the nature of a final judgment is affirmed, without
date of the application but that if the application was not accepted, the money shall special finding as to costs in this instance. So ordered.
be returned. The trite decision of the circuit court of appeal was, "On the conceded
facts of this case, there was no contract to life insurance perfected and the
judgment of the circuit court must be affirmed."

In the case of Cooksey vs. Mutual Life Insurance Co. ([1904], 73 Ark., 117) the
person applying for the life insurance paid and amount equal to the first premium,
but the application and the receipt for the money paid, stipulated that the
insurance was to become effective only when the application was approved and the
policy issued. The court held that the transaction did not amount to an agreement
for preliminary or temporary insurance. It was said:

It is not an unfamiliar custom among life insurance companies in the operation of


the business, upon receipt of an application for insurance, to enter into a contract
with the applicant in the shape of a so-called "binding receipt" for temporary
insurance pending the consideration of the application, to last until the policy be
issued or the application rejected, and such contracts are upheld and enforced
when the applicant dies before the issuance of a policy or final rejection of the
application. It is held, too, that such contracts may rest in parol. Counsel for
appellant insists that such a preliminary contract for temporary insurance was
G.R. No. 3069 January 23, 1907 Nor is there any evidence that Herdman ever undertook to make any parol contract
with Badger for this insurance. There had been some correspondence between the
VIOLA BADGER, plaintiff-appellant,
parties prior to the making of the application on July 5. On that day Herdman,
vs.
writing to Badger in regard to the medical examination, said:
THE NEW YORK LIFE INSURANCE COMPANY, defendant-appellee.
I will send you an official receipt when your remittance reaches the office, and then
Condert Brothers for appellant.
a new examination will not be necessary when the policies are delivered; otherwise
Hartigan, Rohde & Gutierrez for appellee.
this would be necessary.
WILLARD, J.:
After Badger had received the receipt of Herdman for the money sent to him and
On July 5, 1902, William H. Badger made out a written application for a policy of on July 11, he wrote to Herdman, saying:
insurance upon his life for $5,000 in favor of his wife, Harriet Viola Badger. The first
Yours of the 9th instant received. Is the receipt you sent official or not? I do not
premium on this policy amounted to $312.50. Badger sent the application and
wish to take another examination, and so desire an official receipt.
$297.60 to R. E. Herdman, who received the application and the money on the 9th
of July, 1902. xxx xxx xxx

Herdman sent the papers on July 24 to the office of the defendant company in Shall I be obliged to wait until you receive an answer from the office in New York, or
Shanghai, where they were received on August 11. Badger executed a promissory do you have authority to issue policies at the Manila office?
note for $14.90, the balance of the first premium, which was sent to Herdman on
xxx xxx xxx
July 17, 1902. On the 31st of July, Mrs. Badger, acting for her husband, sent to
Herdman $14.90, cash, in payment of said note. Badger died on the 1st day of If my application is accepted does insurance begin July 5, 1902?
August, 1902, of cholera. No policy was ever issued upon his application.
In reply to this letter, Herdman, on July 15, wrote, saying:
The plaintiff brought this action to recover the sum of $5,000, alleging that a
contract of insurance had been made by the company with Badger. Judgment was The receipt I sent you is official, being signed by me as cashier and not personally,
rendered in the court below in favor of the defendant to the effect that no such and of course there will not be another examination required.
contract was ever made, from which judgment the plaintiff appealed.
xxx xxx xxx
The only person who acted in any way for the company in this transaction was
We issue an interim policy from our Shanghai office, which stands until the definite
Herdman. The only evidence in the case to show what his powers were is found in
policy comes from New York. We hope soon to have an advisory board here in
an admission in the answer which states that he was "a special agent and cashier of
Manila, so that we will be entirely free from Shanghai, all our other business being
the defendant company in Manila," and in his evidence, testifying as a witness, he
transacted directly with the home officer at New York.
said that at the time of the trial on September 6, 1905, he was the agency director
of the defendant company in the city of Manila. If your examination is acceptable, your policy will date from July 5, the date of your
application.
The action can not be maintained unless the plaintiff proves a contract between the
company and Badger, made by a person authorized to act for the company. The This evidence shows conclusively that there was no parol agreement between the
authority of this person must, of course, be proven. There is no evidence in the case parties that the insurance had commenced on July 5, 1902. In fact, the claim of the
to show that Herdman had any authority to make any contract, either parol or in appellant reduced to its lowest terms is that the mere signing of an application for
writing, that would bind the company. There is no evidence to show that he had life insurance and the payment of a first premium, without any parol agreement as
any policies in his possession. to when the insurance shall commence, constitutes a contract between the parties
binding from that date. Such a contention as this can not be sustained.
Moreover, there is evidence in the case in addition to that already referred to,
showing that the company expressly refused to be bound until the application had
been accepted either by its office in Shanghai or its office in New York. In the
application which Badger signed on the 5th day of July it is said:

I agree, on behalf of myself and of any person who shall have or claim any interest
in any policy issued under this application, as follows: That inasmuch as only the
officers at the home office of the company in the city of New York have authority to
determine whether or nor a policy shall issue on any application, no statements,
etc., shall be binding on the company.

In the report of the medical examiner there is found this printed statement:

The examiner is requested to send direct to the company in New York City any
information which, for any reason, he prefers not to embody in this report. He can
also mail this report direct to the company if he prefers.

Herdman testifies that when he sent to Badger a receipt for the money paid, it was
on one of two printed blanks, which one he could not say. The court below found
that the receipt was sent upon the blank which contained a reference to the
Shanghai office. Whether it was upon this form of receipt or upon the other one is
of no consequence. In one of them it is stated "that the company shall incur no
liability under the application until it has been received, approved by the resident
board of the company at Shanghai, and a policy issued thereon by the resident
board, and the full premium has actually been paid to and accepted by the
company or its authorized agent during the lifetime and good health of the person
upon whose life the insurance is applied for. The company reserves the absolute
right of disapproval of such application."

The other form contains the statement that "the company shall incur no liability
under the application until it has been received, approved at the house office of the
company, and a policy issued thereon." This is then followed by the words of the
first form. Upon both of these forms are printed the words "conditional receipt."

It seems very clear that no liability was incurred by the company in this case. The
judgment of the court below is accordingly affirmed, with the costs of this instance
against the appellant.

After expiration of twenty days let judgment be entered in accordance herewith


and ten days thereafter the record remanded to the court below for proper action.
So ordered.
G.R. No. L-15895 November 29, 1920 As above suggested, the issue of fact raised by the evidence is whether Herrer
received notice of acceptance of his application. To resolve this question, we
RAFAEL ENRIQUEZ, as administrator of the estate of the late Joaquin Ma.
propose to go directly to the evidence of record.
Herrer, plaintiff-appellant,
vs. The chief clerk of the Manila office of the Sun Life Assurance Company of Canada at
SUN LIFE ASSURANCE COMPANY OF CANADA, defendant-appellee. the time of the trial testified that he prepared the letter introduced in evidence as
Exhibit 3, of date November 26, 1917, and handed it to the local manager, Mr. E. E.
Jose A. Espiritu for appellant.
White, for signature. The witness admitted on cross-examination that after
Cohn, Fisher and DeWitt for appellee.
preparing the letter and giving it to he manager, he new nothing of what became of
MALCOLM, J.: it. The local manager, Mr. White, testified to having received the cablegram
accepting the application of Mr. Herrer from the home office on November 26,
This is an action brought by the plaintiff ad administrator of the estate of the late 1917. He said that on the same day he signed a letter notifying Mr. Herrer of this
Joaquin Ma. Herrer to recover from the defendant life insurance company the sum acceptance. The witness further said that letters, after being signed, were sent to
of pesos 6,000 paid by the deceased for a life annuity. The trial court gave judgment the chief clerk and placed on the mailing desk for transmission. The witness could
for the defendant. Plaintiff appeals. not tell if the letter had every actually been placed in the mails. Mr. Tuason, who
was the chief clerk, on November 26, 1917, was not called as a witness. For the
The undisputed facts are these: On September 24, 1917, Joaquin Herrer made
defense, attorney Manuel Torres testified to having prepared the will of Joaquin
application to the Sun Life Assurance Company of Canada through its office in
Ma. Herrer, that on this occasion, Mr. Herrer mentioned his application for a life
Manila for a life annuity. Two days later he paid the sum of P6,000 to the manager
annuity, and that he said that the only document relating to the transaction in his
of the company's Manila office and was given a receipt reading as follows:
possession was the provisional receipt. Rafael Enriquez, the administrator of the
MANILA, I. F., 26 de septiembre, 1917. estate, testified that he had gone through the effects of the deceased and had
found no letter of notification from the insurance company to Mr. Herrer.
PROVISIONAL RECEIPT Pesos 6,000
Our deduction from the evidence on this issue must be that the letter of November
Recibi la suma de seis mil pesos de Don Joaquin Herrer de Manila como prima dela 26, 1917, notifying Mr. Herrer that his application had been accepted, was prepared
Renta Vitalicia solicitada por dicho Don Joaquin Herrer hoy, sujeta al examen and signed in the local office of the insurance company, was placed in the ordinary
medico y aprobacion de la Oficina Central de la Compañia. channels for transmission, but as far as we know, was never actually mailed and
The application was immediately forwarded to the head office of the company at thus was never received by the applicant.
Montreal, Canada. On November 26, 1917, the head office gave notice of Not forgetting our conclusion of fact, it next becomes necessary to determine the
acceptance by cable to Manila. (Whether on the same day the cable was received law which should be applied to the facts. In order to reach our legal goal, the
notice was sent by the Manila office of Herrer that the application had been obvious signposts along the way must be noticed.
accepted, is a disputed point, which will be discussed later.) On December 4, 1917,
the policy was issued at Montreal. On December 18, 1917, attorney Aurelio A. Until quite recently, all of the provisions concerning life insurance in the Philippines
Torres wrote to the Manila office of the company stating that Herrer desired to were found in the Code of Commerce and the Civil Code. In the Code of the
withdraw his application. The following day the local office replied to Mr. Torres, Commerce, there formerly existed Title VIII of Book III and Section III of Title III of
stating that the policy had been issued, and called attention to the notification of Book III, which dealt with insurance contracts. In the Civil Code there formerly
November 26, 1917. This letter was received by Mr. Torres on the morning of existed and presumably still exist, Chapters II and IV, entitled insurance contracts
December 21, 1917. Mr. Herrer died on December 20, 1917. and life annuities, respectively, of Title XII of Book IV. On the after July 1, 1915,
there was, however, in force the Insurance Act. No. 2427. Chapter IV of this Act
concerns life and health insurance. The Act expressly repealed Title VIII of Book II
and Section III of Title III of Book III of the code of Commerce. The law of insurance In resume, therefore, the law applicable to the case is found to be the second
is consequently now found in the Insurance Act and the Civil Code. paragraph of article 1262 of the Civil Code providing that an acceptance made by
letter shall not bind the person making the offer except from the time it came to his
While, as just noticed, the Insurance Act deals with life insurance, it is silent as to
knowledge. The pertinent fact is, that according to the provisional receipt, three
the methods to be followed in order that there may be a contract of insurance. On
things had to be accomplished by the insurance company before there was a
the other hand, the Civil Code, in article 1802, not only describes a contact of life
contract: (1) There had to be a medical examination of the applicant; (2) there had
annuity markedly similar to the one we are considering, but in two other articles,
to be approval of the application by the head office of the company; and (3) this
gives strong clues as to the proper disposition of the case. For instance, article 16 of
approval had in some way to be communicated by the company to the applicant.
the Civil Code provides that "In matters which are governed by special laws, any
The further admitted facts are that the head office in Montreal did accept the
deficiency of the latter shall be supplied by the provisions of this Code." On the
application, did cable the Manila office to that effect, did actually issue the policy
supposition, therefore, which is incontestable, that the special law on the subject of
and did, through its agent in Manila, actually write the letter of notification and
insurance is deficient in enunciating the principles governing acceptance, the
place it in the usual channels for transmission to the addressee. The fact as to the
subject-matter of the Civil code, if there be any, would be controlling. In the Civil
letter of notification thus fails to concur with the essential elements of the general
Code is found article 1262 providing that "Consent is shown by the concurrence of
rule pertaining to the mailing and delivery of mail matter as announced by the
offer and acceptance with respect to the thing and the consideration which are to
American courts, namely, when a letter or other mail matter is addressed and
constitute the contract. An acceptance made by letter shall not bind the person
mailed with postage prepaid there is a rebuttable presumption of fact that it was
making the offer except from the time it came to his knowledge. The contract, in
received by the addressee as soon as it could have been transmitted to him in the
such case, is presumed to have been entered into at the place where the offer was
ordinary course of the mails. But if any one of these elemental facts fails to appear,
made." This latter article is in opposition to the provisions of article 54 of the Code
it is fatal to the presumption. For instance, a letter will not be presumed to have
of Commerce.
been received by the addressee unless it is shown that it was deposited in the post-
If no mistake has been made in announcing the successive steps by which we reach office, properly addressed and stamped. (See 22 C.J., 96, and 49 L. R. A. [N. S.], pp.
a conclusion, then the only duty remaining is for the court to apply the law as it is 458, et seq., notes.)
found. The legislature in its wisdom having enacted a new law on insurance, and
We hold that the contract for a life annuity in the case at bar was not perfected
expressly repealed the provisions in the Code of Commerce on the same subject,
because it has not been proved satisfactorily that the acceptance of the application
and having thus left a void in the commercial law, it would seem logical to make use
ever came to the knowledge of the applicant.lawph!l.net
of the only pertinent provision of law found in the Civil code, closely related to the
chapter concerning life annuities. Judgment is reversed, and the plaintiff shall have and recover from the defendant
the sum of P6,000 with legal interest from November 20, 1918, until paid, without
The Civil Code rule, that an acceptance made by letter shall bind the person making
special finding as to costs in either instance. So ordered
the offer only from the date it came to his knowledge, may not be the best
expression of modern commercial usage. Still it must be admitted that its
enforcement avoids uncertainty and tends to security. Not only this, but in order
that the principle may not be taken too lightly, let it be noticed that it is identical
with the principles announced by a considerable number of respectable courts in
the United States. The courts who take this view have expressly held that an
acceptance of an offer of insurance not actually or constructively communicated to
the proposer does not make a contract. Only the mailing of acceptance, it has been
said, completes the contract of insurance, as the locus poenitentiae is ended when
the acceptance has passed beyond the control of the party. (I Joyce, The Law of
Insurance, pp. 235, 244.)
G.R. No. 41702 September 4, 1935 policy to Felicidad Estrada upon her payment of the balance of the first year's
annual premium. The agent asked Felicidad Estrada if her nephew was in good
FORTUNATA LUCERO VIUDA DE SINDAYEN, plaintiff-appellant,
health and she replied that she believed so because she had no information that he
vs.
was sick and he thereupon delivered to her the policy.
THE INSULAR LIFE ASSURANCE CO., LTD., defendant-appellee.
On January 20, 1933, the agent learned of the death of Arturo Sindayen and called
Jos. N. Wolfson for appellant.
on Felicidad Estrada and asked her to return the policy. He testified: "pedia a ella
Araneta, Zaragoza and Araneta for appellee.
que me devolviera a poliza para traerla a Manila para esperar la de decision de la
BUTTE, J.: compañia" (t. s. n. p. 19). But he did not return or offer to return the premium paid.
Felicidad Estrada on his aforesaid statement gave him the policy.
This if, an appeal from a judgment of the Court of First Instance of Manila in an
action brought by the plaintiff-appellant as beneficiary to recover P1,000 upon a life On February 4, 1933, under circumstances which it is not necessary to relate here,
insurance policy issued by the defendant on the life of her deceased husband, the company obtained from the beneficiary, the widow of Arturo Sindayen, her
Arturo Sindayen. signature to a legal document entitled "ACCORD, SATISFACTION AND RELEASE"
whereby in consideration of the sum of P40.06 paid to her by a check of the
The essential facts upon which this case turns are not in dispute and may be stated company, she "assigns, releases and forever discharges said Isular Life Assurance
as follows: Co., Ltd., its successors and assigns, of all claims, obligation in or indebtedness
which she, as such beneficiary ever had or now has, hereafter ca, shall, or may
Arturo Sindayen, up to the time of his death on January 19, 1933, was employed as
have, for, upon, or by reason of said policy of life insurance numbered 47710 upon
a linotype operator in the Bureau of Printing at Manila and had been such for
the life of said Arturo Sindayen, the latter now deceased, or arising therefrom or
eleven years prior thereto. He and his wife went to Camiling, Tarlac, to spend the
connected therewith in any manner", which appears in the record as Exhibit A,
Christmas vacation with his aunt, Felicidad Estrada. While there he made a written
attached to the deposition of the notary who executed th fraudulent
application on December 26, 1932, to the defendant Insular Life Assurance Co., Ltd.,
acknowledgment to Exhibit A. The said check for P40.06 was never cashed but
through its agent, Cristobal Mendoza, for a policy of insurance on his life in the sum
returned to the company and appears in the record of this case as Exhibit D.
of P1,000 and he paid to the agent P15 cash as part of the first premium. It was
Thereupon this action was brought to enforce payment of the policy.
agreed with the agent that the policy, when and if issued, should be delivered to his
aunt. Felicidad Estrada, with whom Sindayen left the sum of P26.06 to complete the By the terms of the policy, an annual premium of P40.06 is due on the first day of
payment of the first annual premium of P40.06. On January 1, 1933, Sindayen, who December of each year, the first premium already paid by the insured covering the
was then twenty-nine years of age, was examined by the company's doctor who period from December 1, 1932. It is to December 1, 1933. It is to be noted that the
made a favorable report, to the company. On January 2, 1933, Sindayen returned to policy was not issued and the company assumed no actual risk prior to January 11,
Manila and resumed his work a linotype operator in the Bureau of Printing. On 1933.
January 11, 1933, The company accepted the risk and issued policy No. 47710 dated
back to December 1, 1932, and mailed the same to its agent, Cristobal Mendoza, in The policy contains the following paragraph:
Camiling, Tarlac, for delivery to the insured. On January 11, 1933, Sindayen was at
THE CONTRACT. This Policy and the application herefor constitute the entire
work in the Bureau of Printing. On January 12, he complained of a severe headache
contract between the parties hereto. All statements made by the Insured shall, in
and remained at home. On January 15, he called a physician who found that he was
the absence of fraud, be deemed representations and not warranties, and no such
suffering from acute nephritis and uremia. His illness did not yield to treatment and
statement shall void the Policy unless it is contained in the written application, a
on January 19, 1933, he died.
copy of which is attached to this Policy. Only the President, or the Manager, acting
The policy which the company issued and mailed in Manila on January 11, 1933, jointly with the Secretary or Assistant Secretary (and then only in writing signed by
was received by its agent in Camiling, Tarlac, on January 16, 1933. On January 18, them) have power in behalf of the Company to issue permits, or to modify this or
1933, the agent, in accordance with his agreement with the insured, delivered the any contract, or to extend the time for making any premium payment, and the
Company shall t bound by any promise or representation heretofore hereafter
given by any person other than the above-named officials, and by them only in delivery to the insured unless he is in good health at the time; that this condition
writing and signed conjointly as stated.". precedent goes to the very essence of the contract and cannot be waived by the
agent making delivery of the policy, (Rathbun is. New York Life Insurance Co., 30
The application which the insured signed in Camiling, Tarlac, on December 26, 1932,
Idaho, 34; 165 Pac., 997; American Bankers Insurance Co. vs. Thomas, 53 Okla., 11;
contained among others the following provisions:
154 Pac., 44; Gordon vs. Prudential Insurance Co., 231 Pa., 404; Reliance Life
2. That if this application is accepted and a policy issued in my favor, I bind myself to Insurance Co. vs. Hightower, 148 Ga., 843; 98 S.E., 469.)
accept the same and to pay at least the first year's premium thereon in the City of
On the other hand, a number of American decisions hold that an agent to whom a
Manila.
life insurance policy similar to the one here involved was sent with instructions to
3. That the said policy shall not take effect until the first premium has been paid deliver it to the insured has authority to bind the company by making such delivery,
and the policy has been delivered to and accepted by me, while I am in good health. although the insured was not in good health at the time of delivery, on the theory
that the delivery of the policy being the final act to the consummation of the
4. That the agent taking this application has no authority to make, modify or contract, the condition as to the insurer's good health was waived by the company.
discharge contracts, or to waive any of the Company's right or requirements.". (Kansas City Life Insurance Co. vs. Ridout, 147 Ark., 563; 228 S.W., 55; Metropolitan
Life Insurance Co. vs. Willis, 37 Ind. App., 48; 76 N.E., 560; Grier vs. Mutual Life
The insurance company does not set up any defense of fraud, misconduct or
Insurance Co. of New York, 132 N.C., 543; 44 S.E., 38; Bell vs. Missouri State Life
omission of duty of the insured or his agent, Felicidad Estrada or of the beneficiary.
Insurance Co., 166 Mo. App., 390; 149 S.W., 33.)
In its answer it pleads the "ACCORD, SATISFACTION AND RELEASE" (Exhibit A)
signed by the widow of Arturo Sindayen, the plaintiff-appellant. With respect to A number of these cases go to the of holding that the delivery of the policy by the
Exhibit A, it suffices to say that this release is so inequitable, not to say fraudulent, agent to the insured consummates the contract even though the agent knew that
that we are pleased to note that counsel for the defendant company, on page 51 of the insured was not in good health at the time, the theory being that his knowledge
their brief, state: "si resultara que la poliza aqui en cuestion es valida la apelada is the company's knowledge and his delivery of the policy is the company's delivery;
seria la primera en no dar validez alguno al documento Exhibit A aunque la apelante that when the delivery is made notwithstanding this knowledge of the defect, the
hubiera afirmado que lo otorgo con conocimiento de causa." company is deemed to have waived the defect. Although that appears to be the
prevailing view in the American decisions (14 R.C.L., 900) and leads to the same
It is suggested in appellee's brief that fhere was no delivery of the policy in this case
conclusion, namely, that the act of delivery of the policy in the absence of fraud or
because the policy was not delivered to and accepted by the insured in person.
other ground for recission consummates the insurance, we are inclined to the view
Delivery to the insured in person is not necessary. Delivery may be made by mail or
that it is more consonant with the well known practice of life insurance companies
to a duly constituted agent. Appellee cites no authorities to support its proposition
and the evidence in the present case to rest our decision on the proposition that
and none need be cited to refute it.
Mendoza was authorized by the company to make the delivery of the policy when
We come now to the main defense of the company in this case, namely, that the he received the payment of the first premium and he was satisfied that the insured
said policy never took effect because of paragraph 3 of the application above was in good health. As was well said in the case of MeLaurin vs. Mutual Life
quoted, for at the time of its delivery by the agent as aforesaid the insured was not Insurance Co. (115 S.C., 59; 104 S.E., 327):
in good health. We have not heretofore been called upon to interpret and apply
So much comes from the necessity of the case; the president, the vice-president,
this clause in life insurance application, but identical or substantially identical
and the secretary cannot solicit, or collect, or deliver; they must commit that to
clauses have been construed and applied in a number of cases in the United States
others, and along with it the discretions we have adverted to. . . . The power in the
and the decisions thereon are far from uniform or harmonious. We do not find it
local agent to withhold the policy involves the power to deliver it; there is no
practicable to attempt to determine where the weight of the authority lies and
escape from that conclusion.
propose to resolve this case on its own facts.
But the appellant says, even though the local agent should have concluded that the
There is one line of cases which holds that the stipulation contained in paragraph 3
applicant was in good health, yet, if the fact be the contrary, then the policy never
is in the nature of a condition precedent, that is to say, that there can be no valid
operated. The parties intended to make a contract, and that involved the doing of be required into years afterwards with the view to avoiding the policy on the
everything necessary to carry it into operation, to wit, the acceptance of the ground that it never took effect because of an alleged lack of good health, at the
applicant as a person in good health. They never intended to leave open that one time of delivery. Suppose in the present instance that Sindayen had recovered his
essential element of the contract, when the parties dealth fairly one with the other. health, but was killed in an automobile accident six months after the delivery of the
It is plain, therefore, that upon the facts it is not necessarily a case of waiver or of policy; and that when called on to pay the loss, the company learns of Sindayen's
estoppel, but a case where the local agents, in the exercise of the powers lodged in grave illness on January 18, 1933, and alleges that the policy had never taken effect.
them, accepted the premium and delivered the policy. That act binds their principal, It is difficult to imagine that the insurance company would take such a position in
the defendant. the face of the common belief of the insuring public that when the policy is
delivered, in the absence of fraud or other grounds for rescission, the contract of
Mendoza was duly licensed by the Insurance Commissioner to act as the agent of
insurance is consummated. The insured rests and acts on that faith. So does the
the defendant insurance company. The well known custom of the insurance
insurance company, for that matter, for from the date of delivery of the policy it
business and the evidence in this case prove that Mendoza was not regarded by the
appropriates to its own use the premium paid by the insured. When the policy is
company as a mere conduit or automaton for the performance of the physical act
issued and delivered, in the absence of fraud or other grounds for rescission, it is
of placing the policy in the hands of the insured. If Mendoza were only an
plainly not within the intention of the parties that there should be any questions
automaton then the legally effective delivery of the policy and the consummation
held in abeyance or reserved for future determination that leave the very existence
of the contract occurred when the company expressed its will to release the policy
of the contract in suspense and doubt. If this were not so, the entire business world
by mailing it to its agent, namely, on January 11, 1933. In such a case the agent
which deals so voluminously in insurance would be affected by this uncertainly.
would perform a purely ministerial act and have no discretion. He could do nothing
Policies that have been delivered to the insured are constantly being assigned for
but make unconditional delivery. The legal result would be the same as if the
credit and other purposes. Although such policies are not negotiable instruments
company had mailed the policy on January 11, 1933, to the insured directly using
and are subject to defenses for fraud, it would be a most serious handicap to
the post-office as its conduit for delivery. On January 11, 1933, the insured was in
business if the very existence of the contract remains in doubt even though the
good health performing his regular duties in the Bureau of Printing.
policy has been issued and delivered with all the formalities required by the law. It
But we are not inclined to take such a restrictive view of the agent's authority is therefore in the public interest, for the public is profoundly and generally
because the evidence in the record shows that Mendoza had the authority, given interested in life insurance, as well as in the interest of the insurance companies
him by the company, to withhold the delivery of the policy to the insured "until the themselves by giving certainly and security to their policies, that we are constrained
first premium has been paid and the policy has been delivered to and accepted by to hold, as we, do, that the delivery of the policy to the insured by an agent of the
me (the insured) while I am in good health". Whether that condition had been met company who is authorized to make delivery or without delivery is the final act
or not plainly calls for the exercise of discretion. Granted that Mendoza's decision which binds the company (and the insured as well) in the absence of fraud or other
that the condition had been met by the insured and that it was proper to make a legal ground for rescission. The fact that the agent to whom it has entrusted this
delivery of the policy to him is just as binding on the company as if the decision had duty (and corporation can only act through agents) is derelict or negligent or even
been made by its board of directors. Granted that Mendoza made a mistake of dishonest in the performance of the duty which has been entrusted to him would
judgement because he acted on insufficient evidence as to the state of health of the create a liability of the agent to the company but does not resolve the company's
insured. But it is not charged that the mistake was induced by any misconduct or obligation based upon the authorized acts of the agent toward a third party who
omission of duty of the insured. was not in collusion with the agent.

It is the interest not only the applicant but of all insurance companies as well that Paragraph 4 of the application to the effect "that the agent taking this application
there should be some act which gives the applicant the definite assurance that the has no authority to make, modify or discharge contracts or to waive any of the
contract has been consummated. This sense of security and of peace of mind that company's rights or requirements" is not in point. Mendoza neither waived nor
one's defendants are provided for without risk either of loss or of litigation is the pretended to waive any right or requirement of the company. In fact, his inquiry as
bedrock of life insurance. A cloud will be thrown over the entire insurance business to the state of health of the insured discloses that he was endeavoring to assure
if the condition of health of the the insured at the time of delivery of the policy may himself that this requirement of the company had been satisfied. In doing so, he
acted within the authority conferred on him by his agency and his acts within that
authority bind the company. The company therefore having decided that all the
conditions precedent to the taking effect of the policy had been complied with and
having accepted the premium and delivered the policy thereafter to the insured,
the company is now estopped to assert that it never intended that the policy should
take effect. (Cf. Northwestern Life Association vs. Findley, 29 Tex. Civ. App., 494; 68
S.W, 695; McLaurin vs. Mutual Life Insurance Co., 115 S.C., 59; 104 S.E., 327; 14 Aal.
Jur., par. 12, pages 425-427.)

In view of the premises, we hold that the defendant company assumed the risk
covered by policy No. 47710 on the life of Arturo Sindayen on January 18, 1933, the
date when the policy was delivered to the insured. The judgment appealed from is
therefore reversed with directions to enter judgment against the appellee in the
sum of P1,000 together with interest at the legal rate from and after May 4, 1933,
with costs in both instances against the appellee.
G.R. No. 112329 January 28, 2000 On November 25, 1987, Perez died in an accident. He was riding in a banca which
capsized during a storm. At the time of his death, his application papers for the
VIRGINIA A. PEREZ, petitioner,
additional insurance of P50,000.00 were still with the Gumaca office. Lalog testified
vs.
that when he went to follow up the papers, he found them still in the Gumaca
COURT OF APPEALS and BF LIFEMAN INSURANCE CORPORATION, respondents.
office and so he personally brought the papers to the Manila office of BF Lifeman
YNARES-SANTIAGO, J.: Insurance Corporation. It was only on November 27, 1987 that said papers were
received in Manila.
A contract of insurance, like all other contracts, must be assented to by both
parties, either in person or through their agents and so long as an application for Without knowing that Perez died on November 25, 1987, BF Lifeman Insurance
insurance has not been either accepted or rejected, it is merely a proposal or an Corporation approved the application and issued the corresponding policy for the
offer to make a contract. P50,000.00 on December 2, 1987.4

Petitioner Virginia A. Perez assails the decision of respondent Court of Appeals Petitioner Virginia Perez went to Manila to claim the benefits under the insurance
dated July 9, 1993 in CA-G.R. CV 35529 entitled, "BF Lifeman Insurance policies of the deceased. She was paid P40,000.00 under the first insurance policy
Corporations; Plaintiff-Appellant versus Virginia A. Perez. Defendant-Appellee," for P20,000.00 (double indemnity in case of accident) but the insurance company
which declared Insurance Policy 056300 for P50,000.00 issued by private refused to pay the claim under the additional policy coverage of P50,000.00, the
respondent corporation in favor of the deceased Primitivo B. Perez, null and void proceeds of which amount to P150,000.00 in view of a triple indemnity rider on the
and rescinded, thereby reversing the decision rendered by the Regional Trial Court insurance policy. In its letter' of January 29, 1988 to Virginia A. Perez, the insurance
of Manila, Branch XVI. company maintained that the insurance for P50,000.00 had not been perfected at
the time of the death of Primitivo Perez. Consequently, the insurance company
The facts of the case as summarized by respondent Court of Appeals are not in refunded the amount of P2,075.00 which Virginia Perez had paid.
dispute.
On September 21, 1990, private respondent BF Lifeman Insurance Corporation filed
Primitivo B. Perez had been insured with the BF Lifeman Insurance Corporation a complaint against Virginia A. Perez seeking the rescission and declaration of
since 1980 for P20,000.00. Sometime in October 1987, an agent of the insurance nullity of the insurance contract in question.
corporation, Rodolfo Lalog, visited Perez in Guinayangan, Quezon and convinced
him to apply for additional insurance coverage of P50,000.00, to avail of the Petitioner Virginia A. Perez, on the other hand, averred that the deceased had
ongoing promotional discount of P400.00 if the premium were paid fulfilled all his prestations under the contract and all the elements of a valid
annually.1âwphi1.nêt contract are present. She then filed a counterclaim against private respondent for
the collection of P150,000.00 as actual damages, P100,000.00 as exemplary
On October 20, 1987, Primitivo B. Perez accomplished an application form for the damages, P30,000.00 as attorney's fees and P10,000.00 as expenses for litigation.
additional insurance coverage of P50,000.00. On the same day, petitioner Virginia
A. Perez, Primitivo's wife, paid P2,075.00 to Lalog. The receipt issued by Lalog On October 25, 1991, the trial court rendered a decision in favor of petitioner, the
indicated the amount received was a "deposit."1 Unfortunately, Lalog lost the dispositive portion of which reads as follows:
application form accomplished by Perez and so on October 28, 1987, he asked the
WHEREFORE PREMISES CONSIDERED, judgment is hereby rendered in favor of
latter to fill up another application form.2 On November 1, 1987, Perez was made to
defendant Virginia A. Perez, ordering the plaintiff BF Lifeman Insurance Corporation
undergo the required medical examination, which he passed.3
to pay to her the face value of BF Lifeman Insurance Policy No. 056300, plus double
Pursuant to the established procedure of the company, Lalog forwarded the indemnity under the SARDI or in the total amount of P150,000.00 (any refund made
application for additional insurance of Perez, together with all its supporting papers, and/or premium deficiency to be deducted therefrom).
to the office of BF Lifeman Insurance Corporation at Gumaca, Quezon which office
SO ORDERED.5
was supposed to forward the papers to the Manila office.
The trial court, in ruling for petitioner, held that the premium for the additional (3) Cause of the obligation which is established.
insurance of P50,000.00 had been fully paid and even if the sum of P2,075.00 were
Consent must be manifested by the meeting of the offer and the acceptance upon
to be considered merely as partial payment, the same does not affect the validity of
the thing and the cause which are to constitute the contract. The offer must be
the policy. The trial court further stated that the deceased had fully complied with
certain and the acceptance absolute.
the requirements of the insurance company. He paid, signed the application form
and passed the medical examination. He should not be made to suffer the When Primitivo filed an application for insurance, paid P2,075.00 and submitted the
subsequent delay in the transmittal of his application form to private respondent's results of his medical examination, his application was subject to the acceptance of
head office since these were no longer within his control. private respondent BF Lifeman Insurance Corporation. The perfection of the
contract of insurance between the deceased and respondent corporation was
The Court of Appeals, however, reversed the decision of the trial court saying that
further conditioned upon compliance with the following requisites stated in the
the insurance contract for P50,000.00 could not have been perfected since at the
application form:
time that the policy was issued, Primitivo was already dead.6 Citing the provision in
the application form signed by Primitivo which states that: there shall be no contract of insurance unless and until a policy is issued on this
application and that the said policy shall not take effect until the premium has been
. . . there shall be no contract of insurance unless and until a policy is issued on this
paid and the policy delivered to and accepted by me/us in person while I/We,
application and that the policy shall not take effect until the first premium has been
am/are in good health.9
paid and the policy has been delivered to and accepted by me/us in person while
I/we, am/are in good health The assent of private respondent BF Lifeman Insurance Corporation therefore was
not given when it merely received the application form and all the requisite
the Court of Appeals held that the contract of insurance had to be assented to by
supporting papers of the applicant. Its assent was given when it issues a
both parties and so long as the application for insurance has not been either
corresponding policy to the applicant. Under the abovementioned provision, it is
accepted or rejected, it is merely an offer or proposal to make a contract.
only when the applicant pays the premium and receives and accepts the policy
Petitioner's motion for reconsideration having been denied by respondent court, while he is in good health that the contract of insurance is deemed to have been
the instant petition for certiorari was filed on the ground that there was a perfected.
consummated contract of insurance between the deceased and BF Lifeman
It is not disputed, however, that when Primitivo died on November 25, 1987, his
Insurance Corporation and that the condition that the policy issued by the
application papers for additional insurance coverage were still with the branch
corporation be delivered and received by the applicant in good health, is
office of respondent corporation in Gumaca and it was only two days later, or on
potestative, being dependent upon the will of the insurance company, and is
November 27, 1987, when Lalog personally delivered the application papers to the
therefore null and void.
head office in Manila. Consequently, there was absolutely no way the acceptance of
The petition is bereft of merit. the application could have been communicated to the applicant for the latter to
accept inasmuch as the applicant at the time was already dead. In the case
Insurance is a contract whereby, for a stipulated consideration, one party
of Enriquez vs. Sun Life Assurance Co. of Canada,10 recovery on the life insurance of
undertakes to compensate the other for loss on a specified subject by specified
the deceased was disallowed on the ground that the contract for annuity was not
perils.7 A contract, on the other hand, is a meeting of the minds between two
perfected since it had not been proved satisfactorily that the acceptance of the
persons whereby one binds himself, with respect to the other to give something or
application ever reached the knowledge of the applicant.
to render some service.8 Under Article 1318 of the Civil Code, there is no contract
unless the following requisites concur: Petitioner insists that the condition imposed by respondent corporation that a
policy must have been delivered to and accepted by the proposed insured in good
(1) Consent of the contracting parties;
health is potestative being dependent upon the will of the corporation and is
(2) Object certain which is the subject matter of the contract; therefore null and void.
We do not agree. it may have taken some time for the application papers to reach the main office, in
the case at bar, the same was acted upon less than a week after it was received.
A potestative condition depends upon the exclusive will of one of the parties. For
The processing of applications by respondent corporation normally takes two to
this reason, it is considered void. Article 1182 of the New Civil Code states: When
three weeks, the longest being a month.12 In this case, however, the requisite
the fulfillment of the condition depends upon the sole will the debtor, the
medical examination was undergone by the deceased on November 1, 1987; the
conditional obligation shall be void.
application papers were forwarded to the head office on November 27, 1987; and
In the case at bar, the following conditions were imposed by the respondent the policy was issued on December 2, 1987. Under these circumstances, we hold
company for the perfection of the contract of insurance: that the delay could not be deemed unreasonable so as to constitute gross
negligence.
(a) a policy must have been issued;
A final note. It has not escaped our notice that the Court of Appeals declared
(b) the premiums paid; and Insurance Policy 056300 for P50,000.00 null and void and rescinded. The Court of
Appeals corrected this in its Resolution of the motion for reconsideration filed by
(c) the policy must have been delivered to and accepted by the applicant while he is
petitioner, thus:
in good health.
Anent the appearance of the word "rescinded" in the dispositive portion of the
The condition imposed by the corporation that the policy must have been delivered
decision, to which defendant-appellee attaches undue significance and makes
to and accepted by the applicant while he is in good health can hardly be
capital of, it is clear that the use of the words "and rescinded" is, as it is hereby
considered as a potestative or facultative condition. On the contrary, the health of
declared, a superfluity. It is apparent from the context of the decision that the
the applicant at the time of the delivery of the policy is beyond the control or will of
insurance policy in question was found null and void, and did not have to be
the insurance company. Rather, the condition is a suspensive one whereby the
"rescinded".13
acquisition of rights depends upon the happening of an event which constitutes the
condition. In this case, the suspensive condition was the policy must have been True, rescission presupposes the existence of a valid contract. A contract which is
delivered and accepted by the applicant while he is in good health. There was non- null and void is no contract at all and hence could not be the subject of rescission.
fulfillment of the condition, however, inasmuch as the applicant was already dead
at the time the policy was issued. Hence, the non-fulfillment of the condition WHEREFORE, the decision rendered by the Court of Appeals in CA-G.R. CV No.
resulted in the non-perfection of the contract. 35529 is AFFIRMED insofar as it declared Insurance Policy No. 056300 for
P50,000.00 issued by BF Lifeman Insurance Corporation of no force and effect and
As stated above, a contract of insurance, like other contracts, must be assented to hence null and void. No costs.1âwphi1.nêt
by both parties either in person or by their agents. So long as an application for
insurance has not been either accepted or rejected, it is merely an offer or proposal SO ORDERED.
to make a contract. The contract, to be binding from the date of application, must
have been a completed contract, one that leaves nothing to be done, nothing to be
completed, nothing to be passed upon, or determined, before it shall take effect.
There can be no contract of insurance unless the minds of the parties have met in
agreement.11

Prescinding from the foregoing, respondent corporation cannot be held liable for
gross negligence. It should be noted that an application is a mere offer which
requires the overt act of the insurer for it to ripen into a contract. Delay in acting on
the application does not constitute acceptance even though the insured has
forwarded his first premium with his application. The corporation may not be
penalized for the delay in the processing of the application papers. Moreover, while
G.R. No. 105562 September 27, 1993 c) Notify henceforth it should notify individual beneficiaries designated under any
Group Policy, in the event of the death of insured(s), where the corresponding
LUZ PINEDA, MARILOU MONTENEGRO, VIRGINIA ALARCON, DINA LORENA AYO,
claims are filed by the Policyholder;
CELIA CALUMBAG and LUCIA LONTOK, petitioners,
vs. d) Show cause within ten days why its other responsible officers who have handled
HON. COURT OF APPEALS and THE INSULAR LIFE ASSURANCE COMPANY, this case should not be subjected to disciplinary and other administrative sanctions
LIMITED, respondents. for deliberately releasing to Capt. Nuval the check intended for spouses ALARCON,
in the absence of any Special Power of Attorney for that matter, and for negligence
Mariano V. Ampil, Jr. for petitioners.
with respect to the release of the other five checks.
Ramon S. Caguiao for private respondent.
SO ORDERED. 10

In holding for the petitioners, the Insurance Commission made the following
DAVIDE, JR., J.: findings and conclusions:

This is an appeal by certiorari to review and set aside the Decision of the public After taking into consideration the evidences [sic], testimonial and documentary for
respondent Court of Appeals in CA-G.R. SP No. 229501 and its Resolution denying the complainants and the respondent, the Commission finds that; First: The
the petitioners' motion for reconsideration.2 The challenged decision modified the respondent erred in appreciating that the powers of attorney executed by five (5) of
decision of the Insurance Commission in IC Case the several beneficiaries convey absolute authority to Capt. Nuval, to demand,
No. RD-058. 3 receive, receipt and take delivery of insurance proceeds from respondent Insular
Life. A cursory reading of the questioned powers of authority would disclosed [sic]
The petitioners were the complainants in IC Case No. RD-058, an administrative that they do not contain in unequivocal and clear terms authority to Capt. Nuval to
complaint against private respondent Insular Life Assurance Company, Ltd. obtain, receive, receipt from respondent company insurance proceeds arising from
(hereinafter Insular Life), which was filed with the Insurance Commission on 20 the death of the seaman-insured. On the contrary, the said powers of attorney are
September 1989. 4 They prayed therein that after due proceedings, Insular Life "be couched in terms which could easily arouse suspicion of an ordinary
ordered to pay the claimants their insurance claims" and that "proper man. . . .
sanctions/penalties be imposed on" it "for its deliberate, feckless violation of its
contractual obligations to the complainants, and of the Insurance Code." 5 Insular Second: The testimony of the complainants' rebuttal witness,
Life's motion to dismiss the complaint on the ground that "the claims of Mrs. Trinidad Alarcon, who declared in no uncertain terms that neither she nor her
complainants are all respectively beyond the jurisdiction of the Insurance husband, executed a special power of attorney in favor of Captain Rosendo Nuval,
Commission as provided in Section 416 of the Insurance Code,"6 having been denied authorizing him to claim, receive, receipt and take delivery of any insurance
in the Order of 14 November 1989, 7 it filed its answer on 5 December proceeds from Insular Life arising out of the death of their insured/seaman son, is
1989. 8 Thereafter, hearings were conducted on various dates. not convincingly refuted.

On 20 June 1990, the Commission rendered its decision9 in favor of the Third: Respondent Insular Life did not observe Section 180 of the Insurance Code,
complainants, the dispositive portion of which reads as follows: when it issued or released two checks in the amount of P150,000.00 for the three
minor children (P50,000.00 each) of complainant, Dina Ayo and another check of
WHEREFORE, this Commission merely orders the respondent company to: P40,000.00 for minor beneficiary Marissa Lontok, daughter of another complainant
Lucia Lontok, there being no showing of any court authorization presented or the
a) Pay a fine of FIVE HUNDRED PESOS (P500.00) a day from the receipt of a copy of
requisite bond posted.
this Decision until actual payment thereof;
Section 180 is quotes [sic] partly as follows:
b) Pay and settle the claims of DINA AYO and LUCIA LONTOK, for P50,000.00 and
P40,000.00, respectively;
. . . In the absence of a judicial guardian, the father, or in the latter's absence or Administration (OWWA) benefits from the POEA and to work for the increase of
incapacity, the mother of any minor, who is an insured or a beneficiary under a their PANDIMAN and other benefits arising from the deaths of their husbands/sons.
contract of life, health or accident insurance, may exercise, in behalf of said minor, They were thus made to execute, with the exception of the spouses Alarcon, special
any right, under the policy, without necessity of court authority or the giving of a powers of attorney authorizing Capt. Nuval to, among others, "follow up, ask,
bond where the interest of the minor in the particular act involved does not exceed demand, collect and receive" for their benefit indemnities of sums of money due
twenty thousand pesos . . . . 11 them relative to the sinking of M/V Nemos. By virtue of these written powers of
attorney, complainants-appellees were able to receive their respective death
Insular Life appealed the decision to the public respondent which docketed the case
benefits. Unknown to them, however, the PMSI, in its capacity as employer and
as CA-G.R. SP No. 22950. The appeal urged the appellate court to reverse the
policyholder of the life insurance of its deceased workers, filed with respondent-
decision because the Insurance Commission (a) had no jurisdiction over the case
appellant formal claims for and in behalf of the beneficiaries, through its President,
considering that the claims exceeded P100,000.00,
Capt. Nuval. Among the documents submitted by the latter for the processing of
(b) erred in holding that the powers of attorney relied upon by Insular Life were
the claims were five special powers of attorney executed by complainants-
insufficient to convey absolute authority to Capt. Nuval to demand, receive and
appellees. On the basis of these and other documents duly submitted, respondent-
take delivery of the insurance proceeds pertaining to the petitioners, (c) erred in
appellant drew against its account with the Bank of the Philippine Islands on 27
not giving credit to the version of Insular Life that the power of attorney supposed
May 1986 six (6) checks, four for P200,00.00 each, one for P50,000.00 and another
to have been executed in favor of the Alarcons was missing, and
for P40,00.00, payable to the order of complainants-appellees. These checks were
(d) erred in holding that Insular Life was liable for violating Section 180 of the
released to the treasurer of PMSI upon instructions of
Insurance Code for having released to the surviving mothers the insurance proceeds
Capt. Nuval over the phone to Mr. Mariano Urbano, Assistant Department Manager
pertaining to the beneficiaries who were still minors despite the failure of the
for Group Administration Department of respondent-appellant. Capt. Nuval, upon
former to obtain a court authorization or to post a bond.
receipt of these checks from the treasurer, who happened to be his son-in-law,
On 10 October 1991, the public respondent rendered a decision, 12 the decretal endorsed and deposited them in his account with the Commercial Bank of Manila,
portion of which reads: now Boston Bank.

WHEREFORE, the decision appealed from is modified by eliminating therefrom the On 3 July 1989, after complainants-appellees learned that they were entitled, as
award to Dina Ayo and Lucia Lontok in the amounts of P50,000.00 and P40,000.00, beneficiaries, to life insurance benefits under a group policy with respondent-
respectively. 13 appellant, they sought to recover these benefits from Insular Life but the latter
denied their claim on the ground that the liability to complainants-appellees was
It found the following facts to have been duly established: already extinguished upon delivery to and receipt by PMSI of the six (6) checks
issued in their names.14
It appears that on 23 September 1983, Prime Marine Services, Inc. (PMSI, for
brevity), a crewing/manning outfit, procured Group PoIicy On the basis thereof, the public respondent held that the Insurance Commission
No. G-004694 from respondent-appellant Insular Life Assurance Co., Ltd. to provide had jurisdiction over the case on the ground that although some of the claims
life insurance coverage to its sea-based employees enrolled under the plan. On 17 exceed P100,000.00, the petitioners had asked for administrative sanctions against
February 1986, during the effectivity of the policy, six covered employees of the Insular Life which are within the Commission's jurisdiction to grant; hence, "there
PMSI perished at sea when their vessel, M/V Nemos, a Greek cargo vessel, sunk was merely a misjoinder of causes of action . . . and, like misjoinder of parties, it is
somewhere in El Jadida, Morocco. They were survived by complainants-appellees, not a ground for the dismissal of the action as it does not affect the other reliefs
the beneficiaries under the policy. prayed for." 15 It also rejected Insular Life's claim that the Alarcons had submitted a
special power of attorney which they (Insular Life) later misplaced.
Following the tragic demise of their loved ones, complainants-appellees sought to
claim death benefits due them and, for this purpose, they approached the President On the other hand, the public respondent ruled that the powers of attorney,
and General Manager of PMSI, Capt. Roberto Nuval. The latter evinced willingness Exhibits "1" to "5," relied upon by Insular Life were sufficient to authorize Capt.
to assist complainants-appellees to recover Overseas Workers Welfare
Nuval to receive the proceeds of the insurance pertaining to the beneficiaries. It has been amended by the Family Code 17 which grants the father and mother joint
stated: legal guardianship over the property of their unemancipated common child without
the necessity of a court appointment; however, when the market value of the
When the officers of respondent-appellant read these written powers, they must
property or the annual income of the child exceeds P50,000.00, the parent
have assumed Capt. Nuval indeed had authority to collect the insurance proceeds in
concerned shall be required to put up a bond in such amount as the court may
behalf of the beneficiaries who duly affixed their signatures therein. The written
determine.
power is specific enough to define the authority of the agent to collect any sum of
money pertaining to the sinking of the fatal vessel. Respondent-appellant Hence, this petition for review on certiorari which we gave due course after the
interpreted this power to include the collection of insurance proceeds in behalf of private respondent had filed the required comment thereon and the petitioners
the beneficiaries concerned. We believe this is a reasonable interpretation even by their reply to the comment.
an officer of respondent-appellant unschooled in the law. Had respondent
We rule for the petitioners.
appellant, consulted its legal department it would not have received a contrary
view. There is nothing in the law which mandates a specific or special power of We have carefully examined the specific powers of attorney, Exhibits "1" to "5,"
attorney to be executed to collect insurance proceeds. Such authority is not which were executed by petitioners Luz Pineda, Lucia B. Lontok, Dina Ayo, Celia
included in the enumeration of Art. 1878 of the New Civil Code. Neither do we Calumag, and Marilyn Montenegro, respectively, on 14 May 198618 and uniformly
perceive collection of insurance claims as an act of strict dominion as to require a granted to Capt. Rosendo Nuval the following powers:
special power of attorney. Moreover, respondent-appellant had no reason to doubt
Capt. Nuval. Not only was he armed with a seemingly genuine authorization, he also To follow-up, ask, demand, collect and receipt for my benefit indemnities or sum of
appeared to be the proper person to deal with respondent-appellant being the money due me relative to the sinking of M.V. NEMOS in the vicinity of El Jadida,
President and General Manager of the PMSI, the policyholder with whom Casablanca, Morocco on the evening of February 17, 1986; and
respondent-appellant always dealt. The fact that there was a verbal agreement
To sign receipts, documents, pertinent waivers of indemnities or other writings of
between complainants-appellees and Capt. Nuval limiting the authority of the latter
whatsoever nature with any and all third persons, concerns and entities, upon
to claiming specified death benefits cannot prejudice the insurance company which
terms and conditions acceptable to my said attorney.
relied on the terms of the powers of attorney which on their face do not disclose
such limitation. Under the circumstances, it appearing that complainants-appellees We agree with the Insurance Commission that the special powers of attorney "do
have failed to point to a positive provision of law or stipulation in the policy not contain in unequivocal and clear terms authority to Capt. Nuval to obtain,
requiring a specific power of attorney to be presented, respondents-appellant's receive, receipt from respondent company insurance proceeds arising from the
reliance on the written powers was in order and it cannot be penalized for such an death of the seaman-insured. On the contrary, the said powers of attorney are
act. 16 couched in terms which could easily arouse suspicion of an ordinary man." 19 The
holding of the public respondent to the contrary is principally premised on its
Insofar as the minor children of Dina Ayo and Lucia Lontok were concerned, it ruled
opinion that:
that the requirement in Section 180 of the Insurance Code which provides in part
that: [t]here is nothing in the law which mandates a specific or special power of attorney
to be executed to collect insurance proceeds. Such authority is not included in the
In the absence of a judicial guardian, the father, or in the latter's absence or
enumeration of art. 1878 of the New Civil Code. Neither do we perceive collection
incapacity, the mother, of any minor, who is an insured or a beneficiary under a
of insurance claims as an act of strict dominion as to require a special power of
contract of life, health or accident insurance, may exercise, in behalf of said minor,
attorney.
any right under the policy, without necessity of court authority or the giving of a
bond, where the interest of the minor in the particular act involved does not exceed If this be so, then they could not have been meant to be a general power of
twenty thousand pesos. Such a right, may include, but shall not be limited to, attorney since Exhibits "1" to "5" are special powers of attorney. The execution by
obtaining a policy loan, surrendering the policy, receiving the proceeds of the the principals of special powers of attorney, which clearly appeared to be in
policy, and giving the minor's consent to any transaction on the policy. prepared forms and only had to be filled up with their names, residences, dates of
execution, dates of acknowledgment and others, excludes any intent to grant a q And so it is part of that concept that all inquiries, follow-up, payment of claims,
general power of attorney or to constitute a universal agency. Being special powers premium billings, etc. should always be coursed thru the policyholder?
of attorney, they must be strictly construed.
a Yes that is our practice.
Certainly, it would be highly imprudent to read into the special powers of attorney
q And when you say claim payments should always be coursed thru the policyholder,
in question the power to collect and receive the insurance proceeds due the
do you require a power of attorney to be presented by the policyholder or not?
petitioners from Group Policy No. G-004694. Insular Life knew that a power of
attorney in favor of Capt. Nuval for the collection and receipt of such proceeds was a Not necessarily.
a deviation from its practice with respect to group policies. Such practice was
testified to by Mr. Marciano Urbano, Insular Life's Assistant Manager of the Group q In other words, under a group insurance policy like the one in this case, Insular
Administrative Department, thus: Life could pay the claims to the policyholder himself even without the presentation
of any power of attorney from the designated beneficiaries?
ATTY. CAGUIOA:
xxx xxx xxx
Can you explain to us why in this case, the claim was filed by a certain Capt. Noval
[sic]? WITNESS:

WITNESS: a No. Sir.

a The practice of our company in claim pertaining to group insurance, the ATTY. AMPIL:
policyholder is the one who files the claim for the beneficiaries of the deceased. At
q Why? Is this case, the present case different from the cases which you answered
that time, Capt. Noval [sic] is the President and General Manager of Prime Marine.
that no power of attorney is necessary in claims payments?
q What is the reason why policyholders are the ones who file the claim and not the
WITNESS:
designated beneficiaries of the employees of the policyholders?
a We did not pay Prime Marine; we paid the beneficiaries.
a Yes because group insurance is normally taken by the employer as an employee-
benefit program and as such, the benefit should be awarded by the policyholder to q Will you now tell the Honorable Commission why you did not pay Prime Marine
make it appear that the benefit really is given by the employer. 20 and instead paid the beneficiaries, the designated beneficiaries?

On cross-examination, Urbano further elaborated that even payments, among xxx xxx xxx
other things, are coursed through the policyholder:
ATTY. AMPIL:
q What is the corporate concept of group insurance insofar as Insular Life is
concerned? I will rephrase the question.

WITNESS: q Will you tell the Commission what circumstances led you to pay the designated
beneficiaries, the complainants in this case, instead of the policyholder when as you
a Group insurance is a contract where a group of individuals are covered under one answered a while ago, it is your practice in group insurance that claims payments,
master contract. The individual underwriting characteristics of each individual is not etc., are coursed thru the policyholder?
considered in the determination of whether the individual is insurable or not. The
contract is between the policyholder and the insurance company. In our case, it is WITNESS:
Prime Marine and Insular Life. We do not have contractual obligations with the a It is coursed but, it is not paid to the policyholder.
individual employees; it is between Prime Marine and Insular Life.
q And so in this case, you gave the checks to the policyholder only coursing them primary aim is to provide the employer with a means of procuring insurance
thru said policyholder? protection for his employees and their families at the lowest possible cost, and in so
doing, the employer creates goodwill with his employees, enables the employees to
a That is right, Sir.
carry a larger amount of insurance than they could otherwise, and helps to attract
q Not directly to the designated beneficiaries? and hold a permanent class of employees. 26

a Yes, Sir. 21 In Elfstrom vs. New York Life Insurance Company, 27 the California Supreme Court
explicitly ruled that in group insurance policies, the employer is the agent of the
This practice is usual in the group insurance business and is consistent with the insurer. Thus:
jurisprudence thereon in the State of California — from whose laws our Insurance
Code has been mainly patterned — which holds that the employer-policyholder is We are convinced that the employer is the agent of the insurer in performing the
the agent of the insurer. duties of administering group insurance policies. It cannot be said that the
employer acts entirely for its own benefit or for the benefit of its employees in
Group insurance is a comparatively new form of insurance. In the United States, the undertaking administrative functions. While a reduced premium may result if the
first modern group insurance policies appear to have been issued in 1911 by the employer relieves the insurer of these tasks, and this, of course, is advantageous to
Equitable Life Assurance Society. 22 Group insurance is essentially a single insurance both the employer and the employees, the insurer also enjoys significant
contract that provides coverage for many individuals. In its original and most advantages from the arrangement. The reduction in the premium which results
common form, group insurance provides life or health insurance coverage for the from employer-administration permits the insurer to realize a larger volume of
employees of one employer. sales, and at the same time the insurer's own administrative costs are markedly
reduced.
The coverage terms for group insurance are usually stated in a master agreement or
policy that is issued by the insurer to a representative of the group or to an xxx xxx xxx
administrator of the insurance program, such as an employer. 23 The employer acts
as a functionary in the collection and payment of premiums and in performing The most persuasive rationale for adopting the view that the employer acts as the
related duties. Likewise falling within the ambit of administration of a group policy agent of the insurer, however, is that the employee has no knowledge of or control
is the disbursement of insurance payments by the employer to the over the employer's actions in handling the policy or its administration. An agency
employees. 24 Most policies, such as the one in this case, require an employee to relationship is based upon consent by one person that another shall act in his behalf
pay a portion of the premium, which the employer deducts from wages while the and be subject to his control. It is clear from the evidence regarding procedural
remainder is paid by the employer. This is known as a contributory plan as techniques here that the insurer-employer relationship meets this agency test with
compared to a non-contributory plan where the premiums are solely paid by the regard to the administration of the policy, whereas that between the employer and
employer. its employees fails to reflect true agency. The insurer directs the performance of
the employer's administrative acts, and if these duties are not undertaken properly
Although the employer may be the titular or named insured, the insurance is the insurer is in a position to exercise more constricted control over the employer's
actually related to the life and health of the employee. Indeed, the employee is in conduct.
the position of a real party to the master policy, and even in a non-contributory
plan, the payment by the employer of the entire premium is a part of the total In Neider vs. Continental Assurance Company, 28 which was cited in Elfstrom, it was
compensation paid for the services of the employee. 25 Put differently, the labor of held that:
the employees is the true source of the benefits, which are a form of additional
[t]he employer owes to the employee the duty of good faith and due care in
compensation to them.
attending to the policy, and that the employer should make clear to the employee
It has been stated that every problem concerning group insurance presented to a anything required of him to keep the policy in effect, and the time that the
court should be approached with the purpose of giving to it every legitimate obligations are due. In its position as administrator of the policy, we feel also that
opportunity of becoming a social agency of real consequence considering that the
the employer should be considered as the agent of the insurer, and any omission of Art. 225. The father and the mother shall jointly exercise legal guardianship over
duty to the employee in its administration should be attributable to the insurer. the property of their unemancipated common child without the necessity of a court
appointment. In case of disagreement, the father's decision shall prevail, unless
The ruling in Elfstrom was subsequently reiterated in the cases of Bass vs. John
there is judicial order to the contrary.
Hancock Mutual Life Insurance Co. 29 and Metropolitan Life Insurance Co. vs. State
Board of Equalization.30 Where the market value of the property or the annual income of the child exceeds
P50,000, the parent concerned shall be required to furnish a bond in such amount
In the light of the above disquisitions and after an examination of the facts of this
as the court may determine, but not less than ten per centum (10%) of the value of
case, we hold that PMSI, through its President and General Manager, Capt. Nuval,
the property or annual income, to guarantee the performance of the obligations
acted as the agent of Insular Life. The latter is thus bound by the misconduct of its
prescribed for general guardians.
agent.
It is clear from the said Article that regardless of the value of the unemancipated
Insular Life, however, likewise recognized Capt. Nuval as the attorney-in-fact of the
common child's property, the father and mother ipso jure become the legal
petitioners. Unfortunately, through its official, Mr. Urbano, it acted imprudently
guardian of the child's property. However, if the market value of the property or the
and negligently in the premises by relying without question on the special power of
annual income of the child exceeds P50,000.00, a bond has to be posted by the
attorney. In Strong vs. Repide, 31 this Court ruled that it is among the established
parents concerned to guarantee the performance of the obligations of a general
principles in the civil law of Europe as well as the common law of American that
guardian.
third persons deal with agents at their peril and are bound to inquire as to the
extent of the power of the agent with whom they contract. And in Harry E. Keller It must, however, be noted that the second paragraph of Article 225 of the Family
Electric Co. vs. Rodriguez, 32 this Court, quoting Mechem on Agency, 33 stated that: Code speaks of the "market value of the property or the annual income of the
child," which means, therefore, the aggregate of the child's property or annual
The person dealing with an agent must also act with ordinary prudence and
income; if this exceeds P50,000.00, a bond is required. There is no evidence that the
reasonable diligence. Obviously, if he knows or has good reason to believe that the
share of each of the minors in the proceeds of the group policy in question is the
agent is exceeding his authority, he cannot claim protection. So if the suggestions of
minor's only property. Without such evidence, it would not be safe to conclude
probable limitations be of such a clear and reasonable quality, or if the character
that, indeed, that is his only property.
assumed by the agent is of such a suspicious or unreasonable nature, or if the
authority which he seeks to exercise is of such an unusual or improbable character, WHEREFORE, the instant petition is GRANTED. The Decision of
as would suffice to put an ordinarily prudent man upon his guard, the party dealing 10 October 1991 and the Resolution of 19 May 1992 of the public respondent in CA-
with him may not shut his eyes to the real state of the case, but should either G.R. SP No. 22950 are SET ASIDE and the Decision of the Insurance Commission in IC
refuse to deal with the agent at all, or should ascertain from the principal the true Case No. RD-058 is REINSTATED.
condition of affairs. (emphasis supplied)
Costs against the private respondent.
Even granting for the sake of argument that the special powers of attorney were in
due form, Insular Life was grossly negligent in delivering the checks, drawn in favor
of the petitioners, to a party who is not the agent mentioned in the special power
of attorney.

Nor can we agree with the opinion of the public respondent that since the shares of
the minors in the insurance proceeds are less than P50,000.00, then under Article
225 of the Family Code their mothers could receive such shares without need of
either court appointments as guardian or the posting of a bond. It is of the view
that said Article had repealed the third paragraph of Section 180 of the Insurance
Code. 34 The pertinent portion of Article 225 of the Family Code reads as follows:
G.R. No. L-31845 April 30, 1979 private respondent Ngo Hing. Likewise, petitioner Mondragon handwrote at the
bottom of the back page of the application form his strong recommendation for the
GREAT PACIFIC LIFE ASSURANCE COMPANY, petitioner,
approval of the insurance application. Then on April 30, 1957, Mondragon received
vs.
a letter from Pacific Life disapproving the insurance application (Exhibit 3-M). The
HONORABLE COURT OF APPEALS, respondents.
letter stated that the said life insurance application for 20-year endowment plan is
G.R. No. L-31878 April 30, 1979 not available for minors below seven years old, but Pacific Life can consider the
same under the Juvenile Triple Action Plan, and advised that if the offer is
LAPULAPU D. MONDRAGON, petitioner, acceptable, the Juvenile Non-Medical Declaration be sent to the company.
vs.
HON. COURT OF APPEALS and NGO HING, respondents. The non-acceptance of the insurance plan by Pacific Life was allegedly not
communicated by petitioner Mondragon to private respondent Ngo Hing. Instead,
Siguion Reyna, Montecillo & Ongsiako and Sycip, Salazar, Luna & Manalo for on May 6, 1957, Mondragon wrote back Pacific Life again strongly recommending
petitioner Company. the approval of the 20-year endowment insurance plan to children, pointing out
that since 1954 the customers, especially the Chinese, were asking for such
Voltaire Garcia for petitioner Mondragon.
coverage (Exhibit 4-M).
Pelaez, Pelaez & Pelaez for respondent Ngo Hing.
It was when things were in such state that on May 28, 1957 Helen Go died of
influenza with complication of bronchopneumonia. Thereupon, private respondent
sought the payment of the proceeds of the insurance, but having failed in his effort,
DE CASTRO, J.: he filed the action for the recovery of the same before the Court of First Instance of
The two above-entitled cases were ordered consolidated by the Resolution of this Cebu, which rendered the adverse decision as earlier refered to against both
Court dated April 29, 1970, (Rollo, No. L-31878, p. 58), because the petitioners in petitioners.
both cases seek similar relief, through these petitions for certiorari by way of The decisive issues in these cases are: (1) whether the binding deposit receipt
appeal, from the amended decision of respondent Court of Appeals which affirmed (Exhibit E) constituted a temporary contract of the life insurance in question; and
in toto the decision of the Court of First Instance of Cebu, ordering "the defendants (2) whether private respondent Ngo Hing concealed the state of health and physical
(herein petitioners Great Pacific Ligfe Assurance Company and Mondragon) jointly condition of Helen Go, which rendered void the aforesaid Exhibit E.
and severally to pay plaintiff (herein private respondent Ngo Hing) the amount of
P50,000.00 with interest at 6% from the date of the filing of the complaint, and the 1. At the back of Exhibit E are condition precedents required before a deposit is
sum of P1,077.75, without interest. considered a BINDING RECEIPT. These conditions state that:

It appears that on March 14, 1957, private respondent Ngo Hing filed an application A. If the Company or its agent, shan have received the premium deposit ... and the
with the Great Pacific Life Assurance Company (hereinafter referred to as Pacific insurance application, ON or PRIOR to the date of medical examination ... said
Life) for a twenty-year endownment policy in the amount of P50,000.00 on the life insurance shan be in force and in effect from the date of such medical examination,
of his one-year old daughter Helen Go. Said respondent supplied the essential data for such period as is covered by the deposit ..., PROVIDED the company shall be
which petitioner Lapulapu D. Mondragon, Branch Manager of the Pacific Life in satisfied that on said date the applicant was insurable on standard rates under its
Cebu City wrote on the corresponding form in his own handwriting (Exhibit I-M). rule for the amount of insurance and the kind of policy requested in the application.
Mondragon finally type-wrote the data on the application form which was signed by
D. If the Company does not accept the application on standard rate for the amount
private respondent Ngo Hing. The latter paid the annual premuim the sum of
of insurance and/or the kind of policy requested in the application but issue,
P1,077.75 going over to the Company, but he reatined the amount of P1,317.00 as
or offers to issue a policy for a different plan and/or amount ..., the insurance shall
his commission for being a duly authorized agebt of Pacific Life. Upon the payment
not be in force and in effect until the applicant shall have accepted the policy as
of the insurance premuim, the binding deposit receipt (Exhibit E) was issued to
issued or offered by the Company and shall have paid the full premium thereof. If respondent Ngo Hing over the 20-year endowment life insurance in the amount of
the applicant does not accept the policy, the deposit shall be refunded. P50,000.00 in favor of the latter's one-year old daughter, and with the non-
compliance of the abovequoted conditions stated in the disputed binding deposit
E. If the applicant shall not have been insurable under Condition A above, and the
receipt, there could have been no insurance contract duly perfected between thenl
Company declines to approve the application the insurance applied for shall not
Accordingly, the deposit paid by private respondent shall have to be refunded by
have been in force at any time and the sum paid be returned to the applicant upon
Pacific Life.
the surrender of this receipt. (Emphasis Ours).
As held in De Lim vs. Sun Life Assurance Company of Canada, supra, "a contract of
The aforequoted provisions printed on Exhibit E show that the binding deposit
insurance, like other contracts, must be assented to by both parties either in person
receipt is intended to be merely a provisional or temporary insurance contract and
or by their agents ... The contract, to be binding from the date of the application,
only upon compliance of the following conditions: (1) that the company shall be
must have been a completed contract, one that leaves nothing to be dione, nothing
satisfied that the applicant was insurable on standard rates; (2) that if the company
to be completed, nothing to be passed upon, or determined, before it shall take
does not accept the application and offers to issue a policy for a different plan, the
effect. There can be no contract of insurance unless the minds of the parties have
insurance contract shall not be binding until the applicant accepts the policy
met in agreement."
offered; otherwise, the deposit shall be reftmded; and (3) that if the applicant is not
ble according to the standard rates, and the company disapproves the application, We are not impressed with private respondent's contention that failure of
the insurance applied for shall not be in force at any time, and the premium paid petitioner Mondragon to communicate to him the rejection of the insurance
shall be returned to the applicant. application would not have any adverse effect on the allegedly perfected temporary
contract (Respondent's Brief, pp. 13-14). In this first place, there was no contract
Clearly implied from the aforesaid conditions is that the binding deposit receipt in
perfected between the parties who had no meeting of their minds. Private
question is merely an acknowledgment, on behalf of the company, that the latter's
respondet, being an authorized insurance agent of Pacific Life at Cebu branch office,
branch office had received from the applicant the insurance premium and had
is indubitably aware that said company does not offer the life insurance applied for.
accepted the application subject for processing by the insurance company; and that
When he filed the insurance application in dispute, private respondent was,
the latter will either approve or reject the same on the basis of whether or not the
therefore, only taking the chance that Pacific Life will approve the recommendation
applicant is "insurable on standard rates." Since petitioner Pacific Life disapproved
of Mondragon for the acceptance and approval of the application in question along
the insurance application of respondent Ngo Hing, the binding deposit receipt in
with his proposal that the insurance company starts to offer the 20-year
question had never become in force at any time.
endowment insurance plan for children less than seven years. Nonetheless, the
Upon this premise, the binding deposit receipt (Exhibit E) is, manifestly, merely record discloses that Pacific Life had rejected the proposal and recommendation.
conditional and does not insure outright. As held by this Court, where an agreement Secondly, having an insurable interest on the life of his one-year old daughter, aside
is made between the applicant and the agent, no liability shall attach until the from being an insurance agent and an offense associate of petitioner Mondragon,
principal approves the risk and a receipt is given by the agent. The acceptance is private respondent Ngo Hing must have known and followed the progress on the
merely conditional and is subordinated to the act of the company in approving or processing of such application and could not pretend ignorance of the Company's
rejecting the application. Thus, in life insurance, a "binding slip" or "binding receipt" rejection of the 20-year endowment life insurance application.
does not insure by itself (De Lim vs. Sun Life Assurance Company of Canada, 41 Phil.
At this juncture, We find it fit to quote with approval, the very apt observation of
264).
then Appellate Associate Justice Ruperto G. Martin who later came up to this Court,
It bears repeating that through the intra-company communication of April 30, 1957 from his dissenting opinion to the amended decision of the respondent court which
(Exhibit 3-M), Pacific Life disapproved the insurance application in question on the completely reversed the original decision, the following:
ground that it is not offering the twenty-year endowment insurance policy to
Of course, there is the insinuation that neither the memorandum of rejection
children less than seven years of age. What it offered instead is another plan known
(Exhibit 3-M) nor the reply thereto of appellant Mondragon reiterating the desire
as the Juvenile Triple Action, which private respondent failed to accept. In the
for applicant's father to have the application considered as one for a 20-year
absence of a meeting of the minds between petitioner Pacific Life and private
endowment plan was ever duly communicated to Ngo; Hing, father of the minor intentional or unintentional the concealment entitles the insurer to rescind the
applicant. I am not quite conninced that this was so. Ngo Hing, as father of the contract of insurance (Section 26, Id.: Yu Pang Cheng vs. Court of Appeals, et al, 105
applicant herself, was precisely the "underwriter who wrote this case" (Exhibit H-1). Phil 930; Satumino vs. Philippine American Life Insurance Company, 7 SCRA 316).
The unchallenged statement of appellant Mondragon in his letter of May 6, 1957) Private respondent appears guilty thereof.
(Exhibit 4-M), specifically admits that said Ngo Hing was "our associate" and that it
We are thus constrained to hold that no insurance contract was perfected between
was the latter who "insisted that the plan be placed on the 20-year endowment
the parties with the noncompliance of the conditions provided in the binding
plan." Under these circumstances, it is inconceivable that the progress in the
receipt, and concealment, as legally defined, having been comraitted by herein
processing of the application was not brought home to his knowledge. He must
private respondent.
have been duly apprised of the rejection of the application for a 20-year
endowment plan otherwise Mondragon would not have asserted that it was Ngo WHEREFORE, the decision appealed from is hereby set aside, and in lieu thereof,
Hing himself who insisted on the application as originally filed, thereby implictly one is hereby entered absolving petitioners Lapulapu D. Mondragon and Great
declining the offer to consider the application under the Juvenile Triple Action Plan. Pacific Life Assurance Company from their civil liabilities as found by respondent
Besides, the associate of Mondragon that he was, Ngo Hing should only be Court and ordering the aforesaid insurance company to reimburse the amount of
presumed to know what kind of policies are available in the company for minors P1,077.75, without interest, to private respondent, Ngo Hing. Costs against private
below 7 years old. What he and Mondragon were apparently trying to do in the respondent.
premises was merely to prod the company into going into the business of issuing
endowment policies for minors just as other insurance companies allegedly do.
Until such a definite policy is however, adopted by the company, it can hardly be
said that it could have been bound at all under the binding slip for a plan of
insurance that it could not have, by then issued at all. (Amended Decision, Rollo, pp-
52-53).

2. Relative to the second issue of alleged concealment. this Court is of the firm
belief that private respondent had deliberately concealed the state of health and
piysical condition of his daughter Helen Go. Wher private regpondeit supplied the
required essential data for the insurance application form, he was fully aware that
his one-year old daughter is typically a mongoloid child. Such a congenital physical
defect could never be ensconced nor disguished. Nonetheless, private respondent,
in apparent bad faith, withheld the fact materal to the risk to be assumed by the
insurance compary. As an insurance agent of Pacific Life, he ought to know, as he
surely must have known. his duty and responsibility to such a material fact. Had he
diamond said significant fact in the insurance application fom Pacific Life would
have verified the same and would have had no choice but to disapprove the
application outright.

The contract of insurance is one of perfect good faith uberrima fides meaning good
faith, absolute and perfect candor or openness and honesty; the absence of any
concealment or demotion, however slight [Black's Law Dictionary, 2nd Edition], not
for the alone but equally so for the insurer (Field man's Insurance Co., Inc. vs. Vda
de Songco, 25 SCRA 70). Concealment is a neglect to communicate that which a
partY knows aDd Ought to communicate (Section 25, Act No. 2427). Whether
G.R. No. L-48563 May 25, 1979 On April 20, 1966, Lee See Guat died of lung cancer. Thereafter, the beneficiary of
the two policies, Vicente E. Tang claimed for their face value in the amount of
VICENTE E. TANG, petitioner,
P100,000 which the insurance company refused to pay on the ground that the
vs.
insured was guilty of concealment and misrepresentation at the time she applied
HON. COURT OF APPEALS and PHILIPPINE AMERICAN LIFE INSURANCE
for the two policies. Hence, the filing of Civil Case No. 90062 in the Court of First
COMPANY, respondents.
Instance of Manila which dismissed the claim because of the concealment practised
Ambrosio D. Go for petitioner. by the insured in violation of the Insurance Law.

Ferry, De la Rosa, Deligero Salonga & Associates for private respondent. On appeal, the Court of Appeals, affirmed the decision. In its decision, the Court of
Appeals stated, inter alia: "There is no doubt that she deliberately concealed
material facts about her physical condition and history and/or conspired with
whoever assisted her in relaying false information to the medical examiner,
ABAD SANTOS, J.:
assuming that the examiner could not communicate directly with her."
This is a petition to review on certiorari of the decision of the Court of Appeals (CA-
The issue in this appeal is the application of Art. 1332 of the Civil Code which
G.R. No. 55407-R, June 8, 1978) which affirmed the decision of the Court of First
stipulates:
Instance of Manila in Civil Case No. 90062 wherein the petitioner herein was the
plaintiff and Philippine American Life Insurance Co. the herein respondent was the Art. 1332. When one of the parties is unable to read, or if the contract is in a
defendant. The action was for the enforcement of two insurance policies that had language not understood by him, and mistake or fraud is alleged, the person
been issued by the defendant company under the following circumstances. enforcing the contract must show that the terms thereof have been fully explained
to the former.
On September 25, 1965, Lee See Guat, a widow, 61 years old, and an illiterate who
spoke only Chinese, applied for an insurance on her life for P60,000 with the According to the Code Commission: "This rule is especially necessary in the
respondent Company. The application consisted of two parts, both in the English Philippines where unfortunately there is still a fairly large number of illiterates, and
language. The second part of her application dealt with her state of health and where documents are usually drawn up in English or Spanish." (Report of the Code
because her answers indicated that she was healthy, the Company issued her Policy Commission, p. 136.) Art. 1332 supplements Art. 24 of the Civil Code which provides
No. 0690397, effective October 23, 1965, with her nephew Vicente E. Tang, herein that " In all contractual, property or other relations, when one of the parties is at a
Petitioner, as her beneficiary, disadvantage on account of his moral dependence, ignorance, indigence, mental
weakness, tender age or other handicap, the court must be vigilant for his
On November 15, 1965, Lee See Guat again applied with the respondent Company
protection.
for an additional insurance on her life for P40,000. Considering that her first
application had just been approved, no further medical examination was made but It is the position of the petitioner that because Lee See Guat was illiterate and
she was required to accomplish and submit Part I of the application which reads: spoke only Chinese, she could not be held guilty of concealment of her health
"I/WE HEREBY DECLARE AND AGREE that all questions, statements answers history because the applications for insurance were in English and the insurer has
contained herein, as well as those made to or to be made to the Medical Examiner not proved that the terms thereof had been fully explained to her.
in Part II are full, complete and true and bind all parties in interest under the policy
herein applied for; that there shall be no contract of insurance unless a policy is It should be noted that under Art. 1332 above quoted, the obligation to show that
issued on this application and the fun first premium thereon, according to the mode the terms of the contract had been fully explained to the party who is unable to
of payment specified in answer to question 4D above, actually paid during the read or understand the language of the contract, when fraud or mistake is alleged,
lifetime and good health of the Proposed Insured." Moreover, her answers in Part II devolves on the party seeking to enforce it. Here the insurance company is not
of her previous application were used in appraising her insurability for the second seeking to enforce the contracts; on the contrary, it is seeking to avoid their
insurance. On November 28, 1965, Policy No. 695632 was issued to Lee See Guat performance. It is petitioner who is seeking to enforce them even as fraud or
with the same Vicente E. Tang as her beneficiary. mistake is not alleged. Accordingly, respondent company was under no obligation
to prove that the terms of the insurance contracts were fully explained to the other
party. Even if we were to say that the insurer is the one seeking the performance of
the contracts by avoiding paying the claim, it has to be noted as above stated that
there has been no imputation of mistake or fraud by the illiterate insured whose
personality is represented by her beneficiary the petitioner herein. In sum, Art.
1332 is inapplicable to the case at bar. Considering the findings of both the CFI and
Court of Appeals that the insured was guilty of concealment as to her state of
health, we have to affirm.

WHEREFORE, the decision of the Court of Appeals is hereby affirmed. No special


pronouncement as to costs.

SO ORDERED.

You might also like